MRCP Deck 1 Apr 2023 Flashcards

1
Q

A 23YO F is 5 months pregnant. She reports that she has always had light-brown discolourations, freckling and lumps on her skin but these have worsened in pregnancy. O/E she has multiple light brown pigmented areas on her skin, a number of friable cutaneous skin lesions and axillary freckling. What is the most likely diagnosis?

A) Acanthosis nigricans
B) MEN1
C) MEN2
D) NF1
E) NF2

A

NF1 - results from deletion of the NF1 gene which leads to an increase in CNS tumours. NF1 is associated with less CNS tumours than NF2 but is associated with a mixture of skin features which often increase during puberty and pregnancy.

How well did you know this?
1
Not at all
2
3
4
5
Perfectly
2
Q

A 62 YO F presents with chronic pain following L nephrectomy for renal carcinoma. She has bony pain from metastatic disease and it is planned to start her on meptazinol.

Which of the following is the correct advice with respect to potential constipation?

A) Increase PO fluids only
B) Start regular ispaghula husk
C) Start regular lactulose
D) Start regular macrogol
E) Start regular senna

A

Senna is the initial preferred intervention for preventing opiate induced constipation. Lactulose and movicol can be added to Senna if needed.

How well did you know this?
1
Not at all
2
3
4
5
Perfectly
3
Q

What is the most appropriate management option for a patient in florid pulmonary oedema due to papillary muscle rupture and subsequent MR following an MI?

A) Beta blockers, nitroprusside and diureses
B) Diuresis with furosemide
C) Diuresis and nitroprusside
D) Emergency surgical replacement/repair
E) Thrombolysis

A

The key definitive management is emergency surgery - often valve replacement. Medical management (A) carries a poor prognosis.

How well did you know this?
1
Not at all
2
3
4
5
Perfectly
4
Q

A neutropenic patient has a cough and low grade fever, sparse crepitations and his CXR shows diffuse pulmonary shadowing. No improvement with abx. Sputum shows hyphae but is culture -ve. BC, Aspergillus precipitins and Aspergillus skinprick are all -ve. What is the diagnosis?
A) ABPA
B Aspergilloma
C) Invasive aspergillosis
D) Pneumocystis pneumonia
E) Systemic candidiasis

A

Invasive aspergillosis often presents with a fever, cough with copious amounts of sputum and pleuritic chest pain. It often shows cavitation and non-specific consolidation on CXR with the characteristic halo sign on HRCT. Precipitin and skinprick tests are often negative since the patient is unable to mount an immune response.

How well did you know this?
1
Not at all
2
3
4
5
Perfectly
5
Q

Which of the following cytotoxic agents cause cardiomyopathy as a dose dependent effect?
A) Bleomycin
B) Cytarabine
C) Doxorubicin
D) Methotrexate
E) Paclitaxel

A

Doxorubicin causes a cumulative, dose-dependent decline in L ventricular ejection fraction with a higher incidence of myocardial dysfunction

How well did you know this?
1
Not at all
2
3
4
5
Perfectly
6
Q

Which respiratory condition is most commonly associated with SLE and presents with an unproductive cough and dyspnoea?
A) Asthma
B) Bronchiectasis
C) Bronchiolitis obliterans
D) Fungal infection
E) TB

A

Bronchiolitis obliterans = fibrous scarring of small airways - associated with RA or SLE. Presents as a dry cough and dyspnoea - CXR may be normal or show reticulonodular shadowing. Poor response to steroids and poor prognosis

How well did you know this?
1
Not at all
2
3
4
5
Perfectly
7
Q

Which of the following is associated with confusion, vomiting and causes a yellowish tinge to vision?
A) Amlodipine
B) Aspirin
C) Atorvastatin
D) Digoxin
E) Ramipril

A

Digoxin is associated with disorientation, vomiting, mental confusion, amnesia and depression. It is also associated with a yellow visual field discolouration called xanthopsia.

How well did you know this?
1
Not at all
2
3
4
5
Perfectly
8
Q

A patient presents with hyperventilation and tachycardia. His ABG shows a respiratory alkalosis. Which drug is he likely to have overdosed on?
A) Benzodiazepine
B) TCA
C) Cocaine
D) Paracetamol
E) Theophylline

A

Both theophylline and salicylates are associated with acute respiratory alkalosis. Seizures are common in theophylline overdose which are usually managed with benzodiazepines.

How well did you know this?
1
Not at all
2
3
4
5
Perfectly
9
Q

A patient is found to have reduced factor VIII activity, but at the >5% level. He is due to undergo an extraction of wisdom teeth. What is the most important initial intervention with respect to his management?
A) He should be given desmopressin periprocedure
B) He should be given FFP periprocedure
C) No intervention is needed
D) He should be given Factor VIII periprocedure
E) He should be given cryoprecipitate periprocedure

A

Mild haemophilia (>5%) means the patient is only at significant risk of bleeding associated with trauma or surgery such as tooth extraction. Desmopressin should boost factor VIII activity enough to avoid exogenous administration

How well did you know this?
1
Not at all
2
3
4
5
Perfectly
10
Q

Which of the following statements is the most accurate for a patient with Graves disease and bilateral thyroid eye disease that has relapsed twice after medical treatment?
A) Systemic steroids are of no value in managing the eye disease
B) Subtotal thyroidectomy is the best treatment option for her
C) Stopping smoking will make no difference now that she has established thyroid eye disease
D) Radioiodine treatment is the best treatment option for her
E) Total thyroidectomy is the best treatment option for her

A

Thyroid eye disease may be worsened by radio iodine treatment. Patients with Graves’ hyperthyroidism and active moderate-to-severe or sight-threatening ophthalmopathy should be treated with medications or surgery. even she has related twice with medical treatment, surgery would be the best course of action

How well did you know this?
1
Not at all
2
3
4
5
Perfectly
11
Q

An 11-week pregnant woman presents to hospital with hyperemesis gravidarumm. TFTs are completed and she is found to have an elevated T4 and a low TSH. She has no goitre, thyroid antibodies are negative and she has no symptoms of hyperthyroidism. What is the best management step?
A) Radioiodine
B) Immediate surgical referral
C) Propylthiouracil
D) Carbimazole
E) Observe and wait for normalisation

A

Transient hyperthyroidism of hyperemesis gravidarum can be caused by high levels of hCG - a known stimulator of TSH receptors. Signs and symptoms of hypermetabolism are mild/absent. Most cases often resolve spontaneously in 2-10 weeks.

How well did you know this?
1
Not at all
2
3
4
5
Perfectly
12
Q

A 62 y/o M has a 2M history of fatigue, exertion dyspnoea and abdominal pain. He also has severe pain in his hands (no swellings) and progressive numbness of his feet. O/E red spots are noted on the extensor surfaces of his lower limbs. An XR of the chest shows cardiomegaly. What diagnosis is best suggested by these findings?
A) Dermatomyositis
B) Polyarteritis nodosa
C) Polymyalgia rheumatica
D) Rheumatoid arthritis
E) Sarcoidosis

A

Polyarteritis nodosa (PAN) is a necrotising vasculitis that causes aneurysms of medium sized arteries. Abdominal pain due to mesenteric vasculitis infarction of a viscus. dyspnoea due to pulmonary infiltrates and cardiomegaly, arthralgia, neuropathy and purpuric spots can all occur in this condition.

How well did you know this?
1
Not at all
2
3
4
5
Perfectly
13
Q

Which of the following would cause a rise in the carbon monoxide transfer factor (TLCO)?
A) Emphysema
B) Pulmonary haemorrhage
C) Pulmonary embolism
D) Pneumonia
E) Pulmonary fibrosis

A

Transfer factor = rate at which gas diffuses from alveoli into the blood. Alveolar haemorrhage causes the TLCO to increase due to the enhanced uptake of CO by intra-alveolar haemoglobin.

Raised TLCO = asthma, haemorrhage, L>R shunts, polycythaemia, male gender, exercise

How well did you know this?
1
Not at all
2
3
4
5
Perfectly
14
Q

A patient is found to have a renal angiomyolipoma. What is the most likely underlying diagnosis?
A) Neurofibromatosis
B) Budd-Chiari syndrome
C) Hereditary haemorrhagic telangiectasia
D) Von Hippel-Lindau syndrome
E) Tuberous sclerosis

A

10% of patients with renal angiomyolipomas are found to have underlying tuberous sclerosis whilst most patients with TS are likely to have severe renal angiomyolipomas. TS is an AD condition with neurocutaneous features (similar to neurofibromatosis).

How well did you know this?
1
Not at all
2
3
4
5
Perfectly
15
Q

Which of the following is not associated with an increased risk of developing torsades de pointes?
A) TCA
B) SAH
C) Hypercalcaemia
D) Romano-Ward syndrome
E) Hypothermia

A

Hypercalaemia is associated with QT interval shortening (hypocalcaemia is associated with QT prolongation).

How well did you know this?
1
Not at all
2
3
4
5
Perfectly
16
Q

Which antibiotics should be prescribed to cover for a human bite?
A) Amoxicillin
B) Co-amoxiclav
C) Doxycycline
D) Metronidazole
E) Metronidazole with clarithromycin

A

Human bites, like animal bites, should be treated with co-amoxiclav. Doxycycline + metronidazole is recommended if the patient is penicillin allergic.

How well did you know this?
1
Not at all
2
3
4
5
Perfectly
17
Q

A 57F with PMR taking 10mg prednisolone for the past 5M has a DEXA scan which shows a T-score of -1.5SD. What is the most suitable management?
A) No treatment
B) Vitamin D + calcium supplementation + repeat DEXA in 6M
C) Vitamin D + calcium supplementation
D) Vitamin D + calcium supplementation + hormone replacement
E) Vitamin D + calcium supplementation + oral bisphosphonate

A

Patients with a T score of <-1.5 SD is an indication for a bisphosphonate. This should be co-prescribed with calcium and vitamin D.

How well did you know this?
1
Not at all
2
3
4
5
Perfectly
18
Q

A 25 year old man has a renal biopsy due to worsening renal function. This reveals linear IgG deposits along the basement membrane. What is the most likely diagnosis?
A) SLE
B) IgA nephropathy
C) Minimal change disease
D) Post-streptococcal GN
E) Goodpastures syndrome

A

Goodpastures syndrome = IgG deposits on renal biopsy with anti-GBM antibodies.

How well did you know this?
1
Not at all
2
3
4
5
Perfectly
19
Q

If the mean is 4.6 with an SD of 0.3. Which of the following statements is correct?
A) 95% of the values lie between 4.5 and 4.75
B) 95.4% of the values lie between 4.3 and 4.9
C) 99.7% of the values lie between 4.0 and 5.2
D) 68.3% of the values lie between 4.5 and 4.75
E) 68.3% of the values lie between 4.3 and 4.9

A

68.3% of values of a normally distributed variable lie with 1 SD of the mean. 95.4% of values lie within 2 SD of the mean. 99.7% of values lie within 3 SD of the mean. Within 1.96 SD of the mean lie 95% of the sample values.

How well did you know this?
1
Not at all
2
3
4
5
Perfectly
20
Q

A patient has recently been switched from sodium valproate to lithium. She has presented with bilateral pitting oedema to her knees with her urine protein creatinine ratio of 450mg/mmol and hypoalbuminaemia. What is the most likely underlying pathology?
A) Lithium induced CKD
B) FSGS
C) Minimal change disease
D) Membranous nephropathy
E) IgA nephropathy

A

Minimal change disease is the most common cause of nephrotic syndrome in children and is the most common form of lithium induced nephrotic syndrome. Lithium can also cause FSGS but is les common than lithium-induced minimal change disease.

How well did you know this?
1
Not at all
2
3
4
5
Perfectly
21
Q

A 28 YO F wiht Sjogrens syndrome and SLE has recently began to suffer with lethargy and bony pain. She currently takes hydroxychloroquine and low dose steroids. Her bloods show K 3.1, Bicarb 16, Cr 138 and CCa 2.1. What is the most likely diagnosis?
A) Osteomalacia
B) Renal tubular acidosis type 1
C) Renal tubular acidosis type 2
D) Renal tubular acidosis type 4
E) Secondary hyperparathyroidism

A

RTA type 1 is associated with metabolic acidosis, hypokalaemia and hypocalciuria. It commonly coincides with Sjogrens and SLE.

RTA type 2 is associated with multiple tubular disorders/Fanconi syndrome. It involves impaired retention of bicarbonate in the proximal tubules with resulting systemic acidosis. Hypokalaemia is present.

RTA type 4 (most common) is associated with a reduced response to aldosterone, leading to hyperkalaemic metabolic acidosis in patients with mild chronic renal insufficiency.

How well did you know this?
1
Not at all
2
3
4
5
Perfectly
22
Q

A 57 YO M with advanced cirrhosis presents confused, drowsy and unwell. His BP is 105/65 with a CVP of 14. Urinary catheterisation shows a residual volume of 35ml and he is anuric for the next 2hr. Urine Na is low. USS shows no evidence of obstruction and an empty bladder. Despite IVF, no improvement in urine output is seen. Cr 385, U 8.1. What is the most likely diagnosis?
A) Acute GI haemorrhage
B) ATN secondary to sepsis
C) Hepato-renal syndrome
D) Pre-kidney hypovolaemia
E) SBP

A

Hepato-renal syndrome is a feature of advanced cirrhosis. Urinary Na is useful in differentiating between causes of renal dysfunction. Pre-renal disease is likely if urinary Na <20; intrinsic kidney disease is likely if urinary Na >40; very low urinary Na is a strong indicator of hepatorenal disease. Mx = volume challenge, relief of tense ascites and treatment of any infection. Dialysis may be considered as a last resort.

How well did you know this?
1
Not at all
2
3
4
5
Perfectly
23
Q

A 62M has a rising creatinine following angiography. He feels nauseous and develops HTN with increasing SOB. BP 155/90, HR 85/min, B/L basal crackles consistent with pulmonary oedema. You suspect he may have cholesterol emboli. Which of the following features would be most commonly seen?
A) Low C3 and C4
B) Eosinophili
C) Erythema ab igne
D) Microcytic anaemia
E) Neutropenia

A

Leucocytosis with eosinophilia is common in 80% of patients with cholesterol emboli. The eosinophilia represents a foreign body-type response to cholesterol deposition. Progression to CKD is common.

Low C3 and C4 may be seen in septic emboli (e.g. IE) but not cholesterol emboli.

How well did you know this?
1
Not at all
2
3
4
5
Perfectly
24
Q

A young african woman is diagnosed as having SLE. What is the characteristic epidemiological feature of this condition?
A) First degree relatives have a 25% chance of developing the disease
B) It’s 2x as common in women than men
C) It’s associated with HLA-DR2 and -DR3 in white ethnicity
D) The age of onset is usually >40years
E) The highest incidence is amongst white women

A

There is an increased frequency of HLA-DR2 and -DR3 in white Europeans with SLE

How well did you know this?
1
Not at all
2
3
4
5
Perfectly
25
Q

A 32 F 2-days post partum has suffered two seizures and has been complaining of a severe headache over the previous 24hrs. She received an epidural during delivery. O/E she is drowsy with a BP of 134/87. She has bilateral papilloedema and a temperature of 37.4. What is the most likely diagnosis?
A) Bacterial meningitis
B) Cerebral venous thrombosis
C) Epidural abscess
D) Pre-eclampsia toxaemia
E) SAH

A

Pregnancy and post-partum are both procoagulable states with LP and epidurals also both increasing the risk of cerebral venous thrombosis. The above symptoms are all most consistent with this diagnosis.

How well did you know this?
1
Not at all
2
3
4
5
Perfectly
26
Q

A 25M presents with 1W history of fever and myalgia. He travelled to Chile 8W ago and on return, had a swollen R eyelid for a few weeks. ECG shows non-specific T wave changes in all leads. What is the most likely diagnosis?
A) Echinococcosis
B) Falciparum malaria
C) Schistosomiasis
D) Toxoplasmosis
E) Trypanosomiasis

A

Trypanosoma cruzi causes Chagas disease (common in S America) and is spread by reduvid bugs. They are transmitted by scratching infected faeces of the bug into skin abrasions caused by the bug during blood sucking. In acute trypanosomiasis the patient presents with fever, myalgia, hepatosplenomegaly and myocarditis. Chronically, in small proportion there may be delayed ventricular dilatation with heart failure. Unilateral periorbital oedema and swelling of the eyelid can result from a bug bite around the eyes (Romana’s sign).

How well did you know this?
1
Not at all
2
3
4
5
Perfectly
27
Q

How many repeats of the abnormal triplet sequence would typically be found in a patient with Huntington’s disease?
A) 1-7
B) 7-11
C) 12-33
D) 34-37
E) >37

A

HD is an AD neurodegenerative disorder due to a mutation in CAG trinucleotide in the huntingtin gene on chromosome 4. Normally this triplet sequence contains from about 11-34 repeats - when the number rises >37, HD occurs.

How well did you know this?
1
Not at all
2
3
4
5
Perfectly
28
Q

A 63M presents with a grand mal seizure along with two days of fatigue and drowsiness. He recently underwent chemotherapy for small-cell carcinoma of the bronchus a few months ago. He has an Na of 120 and is diagnosed with SIADH. His fluid intake is restricted but this further falls to 119 with an increased drowsiness. What is the most appropriate management for him?
A) Continue fluid restriction
B) Start dexamethasone
C) Give normal saline 0.9%
D) Give demeclocycline
E) Give hypertonic saline

A

Severe symptomatic hyponatraemia should be treated with IV hypertonic saline (3%) as an initial bolus of 300ml with repeat sodium measurements every 20 mins.

How well did you know this?
1
Not at all
2
3
4
5
Perfectly
29
Q

Anaesthetic recovery time was recorded for 100 patients. Mean and median times for the procedure were 40 and 65 minutes respectively; the SD was 50 and range 95 (20-115 mins). What is the best way of summarising this data for a paper to a medical journal?
A) mean = 40, SD = 50
B) mean = 40, standard error = 5
C) mean = 40, range 20-115
D) median = 65, SD = 50
E) median = 65, range = 20-115

A

This sample population is very skewed as the mean and median times are so disparate. They are both concentrated towards the lower end of the range which is wide. Hence the best way to summarise this skewed data is to use the median and the range as this is a better way to demonstrate an average and the variance in such an asymmetrical population.

How well did you know this?
1
Not at all
2
3
4
5
Perfectly
30
Q

A 53F with long-standing diabetes and gastric dysmotility asks for advice regarding cisapride. Which of the following statements is most accurate regarding cisapride?
A) Decreases PT in patients receiving warfarin
B) Delays gastric emptying time
C) Exacerbation of symptoms of heartburn
D) Relaxation of colonic musculature
E) QT prolongation on the ECG

A

Cisapride is a pro kinetic drug that reduces gastric emptying time and interacts with anticoagulants and increases prothrombin time and bleeding risk. It increases the motility of the entire GI tract. QT abnormalities are reported with therapy and has unfortunately been associated with fatal arrhythmias - leading to restricted use.

How well did you know this?
1
Not at all
2
3
4
5
Perfectly
31
Q

Which of the following agents are most likely to interact with lithium to cause lithium toxicity?
A) Amlodipine
B) Bisoprolol
C) Hydrochlorthiazide
D) Ramipril
E) Valsartan

A

Thiazides carry the greatest risk of lithium toxicity when the medications are co-prescribed, leading to a rapid rise in lithium levels due to a decrease in lithium excretion

How well did you know this?
1
Not at all
2
3
4
5
Perfectly
32
Q

A 45F is admitted with end-stage carcinoma of the breast. She has failed various treatments and you are considering treating her with docetaxel. What is the mode of action of docetaxel?
A) Binding to microtubules
B) Disrupting DNA
C) Inhibiting mitochondrial energy production
D) Inhibiting RNA production
E) Inhibiting ribosome production

A

Docetaxel reversibly binds to microtubules with high affinity, leading to a decrease in the availability of free tubulin, thus preventing mitotic cell activity. Microtubules also accumulate within the cell, increasing the rate of apoptosis.

How well did you know this?
1
Not at all
2
3
4
5
Perfectly
33
Q

Which one of the following features of haemachromatosis may be reversible with treatment?
A) Cardiomyopathy
B) Hypogonadotrophic hypogonadism
C) Diabetes mellitus
D) Arthropathy
E) Liver cirrhosis

A

Cardiomyopathy and skin pigmentation are reversible with treatment. Haemachromatosis is an AR disorder resulting in iron accumulation.

How well did you know this?
1
Not at all
2
3
4
5
Perfectly
34
Q

Which test results indicate Wilsons disease?
A) Low caeruloplasmin, low serum copper, increased 24hr urinary copper excretion
B) High caeruloplasmin, low serum copper, increased 24hr urinary copper excretion
C) Low caeruloplasmin, high serum copper, increased 24hr urinary copper excretion
D) High caeruloplasmin, High serum copper, increased 24hr urinary copper excretion
E) High caeruloplasmin, High serum copper, decreased 24hr urinary copper excretion

A

Low caeruloplasmin, low serum copper, increased 24hr urinary copper excretion - the diagnosis is confirmed by genetic analysis of the ATP7B gene

How well did you know this?
1
Not at all
2
3
4
5
Perfectly
35
Q

Which anti-rheumatic drug should be avoided in patients who have a background of asthma and hypersensitivity to aspirin and NSAIDs causing urticaria and angioedema?
A) Infliximab
B) Leflunomide
C) Methotrexate
D) Rituximab
E) Sulfasalazine

A

Patients who are allergic to aspirin may also react to sulfasalazine. Aspirin hypersensitivity often co-exists with sensitivity to non-steroidal anti-inflammatories and asthma.

How well did you know this?
1
Not at all
2
3
4
5
Perfectly
36
Q

A 25-year-old female has type I diabetes. Her HbA1c is 58 mmol/L. Her blood pressure is 126/68 mmHg. Her BMI is 28 kg/m². She is not keen to increase her total insulin dose. Which of the following adjuncts could you consider to help improve her glycaemic control?
A) Add metformin
B) Switch to mixed insulin regime
C) Add sitagliptin
D) Add exanatide
E) Enrol in supported weight loss programme

A

NICE recommends that adding metformin should be considered in type I diabetics with a BMI > 25 - weight loss is likely to be beneficial but there is greater evidence for benefit with the use of metformin. A mixed insulin regime might be used if a multiple injection basal-bolus regime was not suited to the patient’s lifestyle but is not usually chosen for better glycaemic control.

How well did you know this?
1
Not at all
2
3
4
5
Perfectly
37
Q

A 57-year-old male complains of having very vivid dreams. He reports a 6-week history of having frequent nightmares with extremely disturbing and vivid imagery. You find that he was started on a new medication 6 weeks ago.
Which medication was he likely to have been started on?
A) Nitrate
B) Bisoprolol
C) Amlodipine
D) Verapamil
E) Nicorandil

A

Beta-blockers can cause sleep disturbance.

How well did you know this?
1
Not at all
2
3
4
5
Perfectly
38
Q

A 26M presents 2 weeks following a witnessed seizure which self-resolved. He has no past medical history and has never had a seizure before. CTH NAD, MRI NAD, EEG NAD. Which advice should he be given concerning driving restrictions?
A) The patient is able to drive immediately
B) The patient is unable to drive for at least 6M irrespective of whether AED are started or not
C) The patient is unable to drive for at least 12M irrespective of whether AED are started or not
D) It is at the discretion of the neurologist as to whether this patient can resume driving
E) The patient is unable to drive until they can prove compliance with AEDs

A

Patients cannot drive for 6 months following a first unprovoked or isolated seizure if brain imaging and EEG normal. The minimum interval for which the patient is unable to drive would be extended to 12 months were there evidence for a structural abnormality (e.g. a space-occupying lesion) on imaging which could have caused his seizure, or if there were evidence for epileptiform activity on his EEG.

How well did you know this?
1
Not at all
2
3
4
5
Perfectly
39
Q

64-year-old man presents to the emergency department via ambulance. Four hours earlier, he noticed some speech difficulties. He has a past medical history of hypertension, atrial fibrillation and hypercholesterolaemia and is on regular atorvastatin and ramipril. On examination, he has an expressive dysphasia. There is right-sided hemiplegia, sensory loss and homonymous hemianopia. Urgent CT head and CT angiography excluded intracranial haemorrhage and confirmed occlusion of the proximal anterior circulation. What is the most appropriate management?
A) Apixaban
B) Aspirin 300mg
C) IV thrombolysis
D) IV thrombolysis + mechanical thrombectomy
E) Mechanical thrombectomy

A

Thrombectomy should be offered as soon as possible and within 6 hours of symptom onset, together with intravenous thrombolysis (if within 4.5 hours), to people who have an acute ischaemic stroke and confirmed occlusion of the proximal anterior circulation demonstrated by computed tomographic angiography (CTA) or magnetic resonance angiography (MRA).

How well did you know this?
1
Not at all
2
3
4
5
Perfectly
40
Q

Which one of the following statements regarding macular degeneration is true?
A) Drusen are characteristic of wet macular degeneration
B) Photodynamic therapy is useful in dry macular degeneration
C) Asian ethnicity is a risk factor
D) Male sex is a risk factor
E) Wet macular degeneration carries the worst prognosis

A

Age-related macular degeneration is the most common cause of blindness in the UK. It is characterised by degeneration of retinal photoreceptors that results in the formation of drusen (seen on fundoscopy). It is more common with advancing age and is more common in females. Classically there are two forms of macular degeneration - dry/atrophic macular degeneration (90% of cases, characterised by drusen) and wet/exudative/neovascular macular denegeration (choroidal neovascularisation, carries the worst prognosis)

How well did you know this?
1
Not at all
2
3
4
5
Perfectly
41
Q

For a patient with paroxysmal AF, which part of the electrocardiogram trace should a direct current cardioversion be delivered on for this patient given their arrhythmia?
A) P wave
B) PR interval
C) R wave
D) T wave
E) Unsynchronised shock

A

DC cardioversion for a haemodynamically stable, symptomatic patient with paroxysmal AF is an effective treatment provided the patient has been on anticoagulation for at least 4 weeks. The current should be synchronised with the R wave.

How well did you know this?
1
Not at all
2
3
4
5
Perfectly
42
Q

Which other abnormality of the blood would be most consistent with Polycythaemia vera?
A) Raised ALP
B) Hypokalaemia
C) Thrombocytopaenia
D) Raised ferritin
E) Neutrophilia

A

Polycythaemia vera is a myeloproliferative disorder caused by clonal proliferation of a marrow stem cell leading to an increase in red cell volume, often accompanied by overproduction of neutrophils and platelets.

How well did you know this?
1
Not at all
2
3
4
5
Perfectly
43
Q

Through what mechanism does alcohol intoxication lead to hypoglycaemia?
A) Alcohol causing increased insulin secretion
B) Increased glycogenolysis
C) Increased pancreatic exocrine activity
D) Low levels of carbohydrate content in alcohol decreasing blood sugar levels
E) Promotion of gluconeogenesis

A

Alcohol results in exaggerated insulin secretion causing hypoglycaemia. The mechanism is thought to be due to the effect of alcohol on the pancreatic microcirculation leading to a redistribution of pancreatic blood flow from the exocrine into the endocrine parts and thus increased insulin secretion.

How well did you know this?
1
Not at all
2
3
4
5
Perfectly
44
Q

Which one of the following cytotoxic agents acts by inhibiting dihydrofolate reductase and thymidylate synthesis?
A) Methotrexate
B) Vincristine
C) Bleomycin
D) Cyclophosphamide
E) Doxorubicin

A

Methotrexate - inhibits dihydrofolate reductase and thymidylate synthesis

How well did you know this?
1
Not at all
2
3
4
5
Perfectly
45
Q

What chromosome is the CFTR gene located on?
A) 3
B) 7
C) 11
D) 14
E) 15

A

In the UK 80% of CF cases are due to delta F508 on the long arm of chromosome 7. It is an AR disorder.

How well did you know this?
1
Not at all
2
3
4
5
Perfectly
46
Q

Which of the following is also proven to have a causal link with asbestos exposure?
A) Bronchiectasis
B) T2DM
C) Bronchial carcinoma
D) BCC of the skin
E) IHD

A

Asbestos is well known to increase the risk of mesothelioma, but also increases the risk of bronchial carcinoma, laryngeal cancer and ovarian cancer.

How well did you know this?
1
Not at all
2
3
4
5
Perfectly
47
Q

A 23-year-old woman has been under investigation for fatigue and anaemia for several months. She attends her GP with a rash. On examination, there is a purplish reticulated rash, which is non-blanching, across her limbs and torso. The presence of this rash increases the likelihood of which of the following differential diagnoses?
A) Coeliac disease
B) Borrelia burgdorferi
C) Sarcoidosis
D) Infectious mononucleosis
E) SLE

A

The rash description is classic for livedo reticularis. SLE has an association with livedo reticularis and fits well with the initial presentation.

How well did you know this?
1
Not at all
2
3
4
5
Perfectly
48
Q

A 75F presents with increasing pain and swelling in both ankles, with stiffness and decreased mobility, worsening in the last few months. O/E she also has painful finger joints with nodules at the PIP and DIP. Over the past 2W she has had increasing difficulty passing urine and dysuria. What is the most likely cause of her joint symptoms?
A) Behcets syndrome
B) OA
C) Osteoporosis
D) Reactive arthritis
E) Rheumatoid arthritis

A

The nodules at the PIP and DIP are Bouchards nodes and Heberdens nodes respectively and signify OA of the hands and ankles. The 2W history of dysuria and difficulties passing urine are unrelated.

How well did you know this?
1
Not at all
2
3
4
5
Perfectly
49
Q

Which of the following statements best describes the pharmacological effects of nicorandil?
A) Headache occurs in 2-4% of patients
B) It inhibits ATP-dependent K channels
C) It increases ventricular filling pressures
D) It reverses the hypotensive effect of sildenafil
E) Oral ulceration is a recognised adverse effect

A

Nicorandil is an activator of ATP dependent K channels - its pharmacological actions are to relax smooth muscle in veins and increase venous capacitance, leading to reduced ventricular filling pressures and dilatation of the coronary arterioles. Headache is the most common unwanted effect (35%). Other adverse effects include oral ulceration, flushing and GI disturbance.

How well did you know this?
1
Not at all
2
3
4
5
Perfectly
50
Q

A 67F presents with 10W history of pain affecting the cervical spine, both shoulders, lumbar spine and both hips. Early morning stiffness lasts until lunchtime and she feels very tired. She has lost 5kg of weight in 12 weeks, has a low-grade pyrexia (37.4), B/L knee effusions and R carpal tunnel syndrome. Ix = normocytic anaemia, raised ESR, raised CRP, weakly positive RF, serum immunoglobulins and protein electrophoresis show a polyclonal increase in gammaglobulins and elevated a1 and a2 globulins. What is the most likely diagnosis?
A) Paraneoplastic syndrome
B) PMR
C) Polymyositis
D) Rheumatoid arthritis
E) Temporal arteritis

A

PMR usually affects elderly white females - symmetrical arthritis with a polyclonal increase in globulins (marker of inflammation). Early morning stiffness, particularly of the hip and shoulder girdle with associated fatigue can last 2-3 hours each morning. Transient synovitis can occur and carpal tunnel may be associated in up to 15% of cases

How well did you know this?
1
Not at all
2
3
4
5
Perfectly
51
Q

A 20M is referred due to hypogonadism and infertility. He went through normal puberty and there is no significant family history. O/E he is tall with gynaecomastia and small testes. He has a normal sense of smell. Which condition would be high on your list of differentials?
A) Fragile X synrome
B) Kallmann syndrome
C) Klinefelter syndrome
D) Marfan syndrome
E) XYY syndrome

A

Klinefelter syndrome occurs due to the presence of an additional X chromosome. It is the most common cause of male hypogonadism and infertility. Affected men tend to be tall and have an increased risk of mild intellectual impairment. They may also have an increased risk of osteoporotic fracture due to low testosterone.

How well did you know this?
1
Not at all
2
3
4
5
Perfectly
52
Q

Which cardiac lesion is most likely to be prone to IE?
A) Aortic regurg
B) ASD
C) Pulmonary valve regurg
D) Tricuspid regurg
E) Tricuspid stenosis

A

Aortic regurgitation. Endocarditis commonly affects LHS>RHS. IE most commonly affects mitral>aortic>aortic+mitral>tricuspid>pulmonary - if the valve is already abnormal then the likelihood of infection is greater.

How well did you know this?
1
Not at all
2
3
4
5
Perfectly
53
Q

What is the best diagnostic test for CJD?
A) CT brain
B) EEG
C) LP
D) MRI
E) SPECT scan

A

LP has 95.8% diagnostic sensitivity and 100% specificity. The gold standard used to be MRI brain but this has been superseded by LP results.

How well did you know this?
1
Not at all
2
3
4
5
Perfectly
54
Q

The risk of complications in the control arm is 2.4% an the risk of complications in the treatment arm is 1.8%. What is the absolute risk reduction associated with the new treatment?
A) 0.6%
B) 1.8%
C) 2.4%
D) 25%
E) 33%

A

The absolute risk reduction is the difference in risk between the control and the intervention groups - 0.6%

How well did you know this?
1
Not at all
2
3
4
5
Perfectly
55
Q

Which of the following is the best advice for a patient in the post-radioiodine period?
A) It is safe to try and conceive after 3M
B) She should not have close contact with young children and pregnant women for 10D post treatment
C) There is no need to monitor her TSH level
D) She will never need further doses of radioiodine
E) She can be in close contact with children within 1 day

A

Although the amount of radioiodine during treatment is small, it is recommended to wait approx 2 weeks before having close contact with children and pregnant women. It is recommended to wait at least 6M before attempting to conceive.

How well did you know this?
1
Not at all
2
3
4
5
Perfectly
56
Q

When considering dietary protein, which of the following best describes the site of amino acid absorption?
A) The caecum
B) The proximal stomach
C) The jejunum
D) The distal stomach
E) The duodenum

A

Most absorption occurs in the jejunum after degradation in the duodenum by pancreatic proteases.

How well did you know this?
1
Not at all
2
3
4
5
Perfectly
57
Q

Which of the following pathological changes is a characteristic feature of the Wernicke-Korsakoff syndrome?
A) Cerebellar atrophy
B) Dilatation of the IIIrd venricle
C) Neuronal loss in the mamillary bodies
D) Demyelination in the pons medulla
E) Microvascular lesions in the cortex

A

The pathological changes are symmetrical lesions in the walls of the third ventricle and periaqueductal grey matter, with subsequent atrophy of the mamillary bodies.

How well did you know this?
1
Not at all
2
3
4
5
Perfectly
58
Q

What is the most appropriate way to confirm H. Pylori eradication?
A) 13C urea breath test
B) Endoscopy and biopsy for sensitivities
C) Endoscopy and urease test
D) Faecal H. pylori antigen
E) H. pylori serology

A

Confirmation of H. Pylori eradication occurs 4W post completion of antibiotics and whilst off PPIs for 2W. Non-invasive tests should be used unless the patient has complex peptic ulcer disease or a MALToma. The most specific and sensitive non-invasive test is the 13C urea breath test.

How well did you know this?
1
Not at all
2
3
4
5
Perfectly
59
Q

What are you most likely to hear on auscultation of a patient with LBBB and a previous MI?
A) Ejection systolic murmur
B) Fixed split second heart sound
C) Pan-systolic murmur
D) Quiet 2nd heart sound
E) Reverse split 2nd heart sound

A

Causes of a reverse split 2nd heart sound include: aortic stenosis, HOCM and IHD with LBB

How well did you know this?
1
Not at all
2
3
4
5
Perfectly
60
Q

Which of the following is the underlying cause of thyroid eye disease?
A) Apoptosis of fibroblasts
B) Glycosaminoglycan (GAG) deposition
C) Iatrogenic hypothyroidism
D) Inflammatory optic neuritis
E) IgG deposition

A

Fibroblasts are stimulated by anti-TSH receptors and activated T cells and lead to GAG deposition. GAG is hydrophilic so draws in water from surrounding tissues and leads to significant oedema. Visual changes result from pressure being placed on the optic nerve.

How well did you know this?
1
Not at all
2
3
4
5
Perfectly
61
Q

A 64Y/O smoker has a 12hr history of weakness in his legs and inability to pass urine. He is hypertensive with flaccid paralysis of the lower limbs. Pain and temperature sensation are impaired to T7 but joint position is unaffected in the lower limbs. What is the most likely diagnosis?
A) Acute transverse myelitis
B) Anterior spinal artery occlusion
C) Cauda equina syndrome
D) Cervical spondylosis with acute decompensation
E) Spinal tumour

A

Branches of the anterior spinal artery supply the anterior 2/3 of the spinal cord, with the posterior spinal arteries supplying the posterior third. There are segments of the cord in the watershed area between the branches T2-T4 which are vulnerable to ischaemia. It can present with sudden back pain with B/L flaccid weakness and dissociated sensory loss with impaired pinprick and temperature sensation below the level of the infarct. Proprioception and vibration sense are typically spared as they are conducted in the posterior columns.

Cauda equina could cause weakness in both legs but the sensory level at T7 localises the lesion to the spinal cord and not the more distal cauda equina.

How well did you know this?
1
Not at all
2
3
4
5
Perfectly
62
Q

A 55F presents with abdominal pain on BG of weight loss and night sweats. O/E there is significant splenomegaly and mild hepatomegaly but no clinical enlargement of the peripheral lymph nodes. FBC: RBC 89, WCC 5.4x10(9), platelets 470m MCV 85, reticulocytes 2.4%. Blood film shows elliptocytes, occasional myelocytes and nucleated RBC. Serum LDH 1256. What is the most likely diagnosis?
A) CML
B) Essential thrombocythaemia
C) Megaloblastic anaemia
D) Myelofibrosis
E) Non-Hodgkins lymphoma

A

Significant splenomegaly + mild hepatomegaly, normochromic anaemia and a relatively normal WCC. Blood film shows early WBC and RBC precursors (leukoerythroblastic blood picture). The significantly enlarged spleen and normal WCC strongly suggests myelofibrosis.

CML is the other differential for a significantly enlarged spleen but this would have a very elevated WCC.

How well did you know this?
1
Not at all
2
3
4
5
Perfectly
63
Q

A 71F is admitted for collapse shortly after complaining of a severe headache. She has a history of HTN with a BP 195/110. Her pupils are pinpoint, but just reactive to high-intensity light. She is comatose and gazing forward but there is no movement of the eyes on head turning. Tone is increased with plantars upgoing. What is the most likely cause of her stroke?
A) Lacunar haemorrhage
B) Lateral medullary syndrome
C) Pontine haemorrhage
D) Left middle cerebral artery infarct
E) Thalamic haemorrhage

A

The features that point towards a pontine haemorrhage are the loss of the dolls eye reflex (confirms a nuclear rather than supranuclear palsy) and B/L pinpoint pupils which are often features of pontine lesions.

Lateral medullary syndrome is associated with altered facial sensation ipsilaterally and altered limb sensation contralaterally. Thalamic haemorrhage is associated with contralateral sensory loss and ataxia.

How well did you know this?
1
Not at all
2
3
4
5
Perfectly
64
Q

A 64M on peritoneal dialysis due to T2DM is admitted with a NOF. Serum Ca 2.91, phosphate 2.2, PTH 0.4. He takes alfacalcidol and calcium carbonate TDS. What would a bone biopsy show?
A) Adynamic bone disease
B) Aluminium deposition at the osteoid bon interface
C) An increase in plasma cells
D) Osteitis fibrosa cystica
E) Osteoporosis

A

Adynamic bone disease is most prevalent in diabetic patients and those on peritoneal dialysis, and is associated with a higher risk of NOF. There is a tendency towards hypercalcaemia as the bone loses its capacity to buffer serum calcium. It can be caused by overtreatment with alfacalcidol. Histologically it is characterised by a reduction in both bone formation and resorption with thin osteoid seams, little active mineralisation and few osteoclasts.

How well did you know this?
1
Not at all
2
3
4
5
Perfectly
65
Q

A 24F with a history of anorexia is found to have sores and blisters over the legs and bony prominences which did not heal, thinning hair and a sore mouth. She is underweight with fine lanugo hair over her body and glossitis. Which nutrient deficiency is likely to have caused her symptoms?
A) Copper
B) Folic acid
C) Vitamin A
D) Vitamin C
E) Zinc

A

Zinc deficiency is common in AN patients and causes poor wound healing, hair loss and glossitis. Zinc is common in red meat and shellfish and may be present in some cereals and pulses - it is important to replace as it impairs an individuals ability to gain weight.

Vitamin C deficiency can present similarly but is uncommon in AN due to its abundance in fresh fruit and vegetables.

How well did you know this?
1
Not at all
2
3
4
5
Perfectly
66
Q

Which of the following is most likely to prevent curative surgery for lung carcinoma?
A) FEV1 of 1.6L
B) Hypercalcaemia
C) Local invasion by primary tumour through the chest wall
D) Malignant pleural effusions
E) Mediastinal lymph nodes enlarged on CT

A

Malignant pleural effusions usually imply widely disseminated disease and any treatment is often palliative.

How well did you know this?
1
Not at all
2
3
4
5
Perfectly
67
Q

A 32M presents with gradual onset B/L UL weakness especially affecting the hands and wrists. His only PMHx includes a rear shunt car accident a few months earlier. There is normal vibration and position sense in both UL with pain and temperature sensation lost over the shoulders and upper arms. There is distal power weakness affecting both hands and wrists. Which of the following is the most likely diagnosis?
A) Cervical myelopathy
B) Intramedullary spinal cord tumour
C) MS
D) Transverse myelitis
E) Syringomyelia

A

The gradual onset of dissociated sensory loss in a shawl-like distribution over the shoulders, coupled with distal weakness affecting both hands fits best with syringomyelia. It may also begin after an episode of trauma such as a traffic accident.

How well did you know this?
1
Not at all
2
3
4
5
Perfectly
68
Q

A 20M with epilepsy presents to ED with a 3 day history of cough and increasing SOB. He has foul smelling sputum. He had a generalised T-C seizure one week ago. O/E his temperature is 40 degrees, he has R sided creps and CXR shows right middle lobe infiltration. What is the most likely cause?
A) Mycoplasma pneumonia
B) Chemical pneumonitis
C) Pneumonia due to Gram -ve aerobes
D) Pneumonia due to Gram +ve aerobes
E) Pneumonia due to anaerobes

A

Anaerobic pleuropulmonary infections include aspiration pneumonia, necrotising pneumonitis, lung abscess and empyema. It is likely this patient aspirated following his seizure which has led to an anaerobic pneumonia. Patients will often mention a revolting taste and smell of their sputum.

How well did you know this?
1
Not at all
2
3
4
5
Perfectly
69
Q

Which of the following medications is most likely to cause significant warfarin resistance?
A) Amlodipine
B) Azathioprine
C) COCP
D) Mesalazine
E) Prednisolone

A

It is a recognised effect that azathioprine causes warfarin resistance but the exact mechanism is unclear ?induction of hepatic enzymes enhancing warfarin clearance.

How well did you know this?
1
Not at all
2
3
4
5
Perfectly
70
Q

A 50M chronic alcoholic presents with a persistent skin rash on his hands, arms, neck and face. The rash is red-brown in colour, symmetrical and scaly. He also complains of a poor appetite, nausea and diarrhoea. Which vitamin deficiency is most likely to have caused his symptoms?
A) Niacin
B) Thiamine
C) Vitamin B6
D) Folic acid
E) Zinc

A

Niacin (B3) deficiency is characterised by dermatitis, diarrhoea and dementia, a condition known as pellagra.

How well did you know this?
1
Not at all
2
3
4
5
Perfectly
71
Q

A 71M presents with severe pain around his right eye and vomiting. O/E the right eye is red and decreased visual acuity is noted. Which one of the following options is the most appropriate initial treatment?
A) Topical pilocarpine and PO steroids
B) Topical pilocarpine and topical steroids
C) Topical steroids
D) Topical pilocarpine and IV acetazolamide
E) Topical steroids and IV acetazolamide

A

Glaucoma is a disorder characterised by optic neuropathy as a result of raised IOP. In acute angle-closure glaucoma (AACG) there is a rise in IOP secondary to an impairment of aqueous outflow.

Features: severe ocular pain, headache, decreased visual acuity, worse with mydriasis
hard and red-eye, haloes around lights, semi-dilated non-reacting pupil. Emergency medical treatment is required to lower the IOP with more definitive surgical treatment given once the acute attack has settled. Medical treatment includes IV acetyzolamide and a combination of eye drops - a direct parasynthomimetic (pilocarpine), a b-blocker (timolol), and an alpha-2-agonist (apraclonidine). Definitive treatment is with laser peripheral iridotomy.

How well did you know this?
1
Not at all
2
3
4
5
Perfectly
72
Q

Which one of the following factors indicates a poor prognosis in patients with multiple sclerosis?
A) Relapsing-remitting disease
B) Presence of sensory symptoms
C) Young age of onset
D) Male sex
E) Long interval between first 2 relapses

A

Good prognosis features
-Female sex
-Age: young age of onset (i.e. 20s or 30s)
-Relapsing-remitting disease
-Sensory symptoms only
-Long interval between first two relapses
complete recovery between relapses

A typical patient carries a better prognosis than an atypical presentation.

How well did you know this?
1
Not at all
2
3
4
5
Perfectly
73
Q

A 12M presents with a purpuric rash on the extensor surfaces of his lower legs. He has a history of abdominal pain and an urticarial rash and has blood ++ in his urine. What would be the likely finding on renal biopsy?
A) Linear IgG deposits
B) No change
C) Sclerosis within the glomerulus
D) Mesangial hypercellularity
E) Basement membrane thickening

A

Henoch-Schonlein purpura (HSP) is an IgA mediated small vessel vasculitis. There is a degree of overlap with IgA nephropathy (Berger’s disease) but is usually seen in children following an infection. It is characterised by mesangial hypercellularity.

How well did you know this?
1
Not at all
2
3
4
5
Perfectly
74
Q

A 65M presents with 2W worsening shortness of breath, worse at night-time. Hx of well controlled HTN and childhood rheumatic fever. O/E you note a pulsation of the patients nail bed with a BP 170/56. What would you be most likely to hear on auscultation?
A) Continuous machinery murmur
B) Early diastolic murmur
C) Ejection systolic murmur
D) Late systolic murmur
E) Pan systolic murmur

A

Aortic regurgitation can present with symptoms including dyspnoea, orthopnoea, and paroxysmal nocturnal dyspnoea. He is at risk of aortic regurgitation due to his history of rheumatic fever. Further signs are the wide pulse pressure (114mg) and nail-bed pulsation (Quincke’s sign).

Aortic regurgitation is associated with an early diastolic murmur, loudest on expiration.

How well did you know this?
1
Not at all
2
3
4
5
Perfectly
75
Q

A 34F with no past medical history presents following a fainting episode. She has had several recently and also has gradually worsening exertional breathlessness, which limits her from walking up a flight of stairs. An ECG and 24-hour tape have revealed right axis deviation. O/E she has normal heart and breath sounds, with mild peripheral oedema and a right ventricular heave. What test will be most important in deciding how this patient is managed?
A) CTCA
B) CTPA
C) HRCT
D) TTE
E) Vasodilator testing

A

Vasodilator testing is the cornerstone test in determining the management of pulmonary artery hypertension (PAH).

It usually affects 30-60YO women between and presents with exertional breathlessness and signs of right heart failure (e.g. right axis deviation suggests right ventricular hypertrophy, and peripheral oedema suggests right heart failure). The test involves giving vasodilator agents (e.g. nitric oxide, epoprostenol) and looking for a reduction in pulmonary arterial pressure (usually, a 10mmHg drop is classified as positive). 10-15% of PAH patients will be positive in this test, which suggests a likely benefit from calcium channel blockers in managing the condition. Those with a negative test are treated with prostacyclin analogs, endothelin receptor antagonists or phosphodiesterase inhibitors.

How well did you know this?
1
Not at all
2
3
4
5
Perfectly
76
Q

A patient diagnosed with chronic primary hyperparathyroidism was treated by parathyroidectomy and you are seeing him in clinic as a follow-up. His PTH and phosphate are now both in range but his calcium is found to be 1.7. What is the most likely explanation for this?
A) Osteomalacia
B) Hungry bone syndrome
C) Rickets
D) Scurvy
E) Parathyroid adenoma

A

Hungry bone syndrome rare condition that occurs as a result of a sudden drop in previously high parathyroid hormone levels. It is a significant complication of parathyroidectomy’s and causes hypocalcaemia.

How well did you know this?
1
Not at all
2
3
4
5
Perfectly
77
Q

On auscultation it is noted that the pulmonary component of the second heart sound occurs before the aortic. Which one of the following is associated with this finding?
A) Pulmonary stenosis
B) LBBB
C) RBBB
D) ASD
E) Deep inspiration

A

S2 is caused by the closure of the aortic valve (A2) closely followed by that of the pulmonary valve (P2).

Loud S2
-Hypertension: systemic (loud A2) or pulmonary (loud P2)

Soft S2
-Aortic stenosis

Fixed split S2
-Atrial septal defect

Widely split S2
-Deep inspiration
-RBBB
-Pulmonary stenosis

Reversed (paradoxical) split S2
-LBBB
-Severe aortic stenosis
-WPW type B (causes early P2)
-Patent ductus arteriosus

How well did you know this?
1
Not at all
2
3
4
5
Perfectly
78
Q

A 45F has blood tests taken and a urine sample dipped at a screening appointment for a clinical trial she has signed up for. All her results are normal except for an eGFR of 62 and +proteinuria. What is the most accurate description of these results?
A) CKD1
B) CKD2
C) CKD3a
D) CKD3b
E) No CKD

A

1 Greater than 90 ml/min, with some sign of kidney damage on other tests (if all the kidney tests* are normal, there is no CKD)

2 60-90 ml/min with some sign of kidney damage (if kidney tests* are normal, there is no CKD)

3a 45-59 ml/min

3b 30-44 ml/min

4 15-29 ml/min

5 Less than 15 ml/min - dialysis or a kidney transplant may be needed

How well did you know this?
1
Not at all
2
3
4
5
Perfectly
79
Q

A 72M is reviewed for management of chronic heart failure, established on bisoprolol, ramipril and eplerenone. He has ongoing symptoms of breathlessness and swelling of the legs. Obs are within range, ECG shows NSR and TTE shows LVEF of 30% with no evidence of valvular disease. What alteration to the patient’s medication is most appropriate?
A) Add hydralazine
B) Add spironolactone
C) Stop bisoprolol and start carvedilol
D) Stop ramipril and start ivabradine
E) Stop ramipril and start sacubitril-valsartan

A

Sacubitril-valsartan would be the most appropriate to commence as the patient has an LVEF <35%, is symptomatic and already established on an ACE-i and beta-blocker. This drug combines the neprilysin-inhibitor sacubitril with the angiotensin receptor blocker (ARB) valsartan. As sacubitril-valsartan contains an ARB it should only be commenced following a wash-out period of any ACEi or ARB a patient is already established on (in this case the patient’s ramipril should be stopped).

Hydralazine may be considered as an alternative third agent in the management of chronic heart failure. However, it must be used in combination with nitrates.

Spironolactone is an aldosterone antagonist used as a second-line agent . This patient has already been established on eplerenone (aldosterone antagonist), and remains symptomatic therefore another class of drug must be commenced.

Ivabradine is another third-line agent that can be used, however, the patient must have a resting heart rate of at least 75/min to be commenced on this drug.

OVERVIEW (with reduced EF)
1st line - ACEi + b-blocker
2nd line - aldosterone antagonist
3rd line - ivabradine, sacubitril valsartan, hydralazine in combo with nitrate, digoxin, cardiac resynchronisation therapy

How well did you know this?
1
Not at all
2
3
4
5
Perfectly
80
Q

A 37Fpatient presents with 5D of altered personality, visual and auditory hallucinations. On palpation, a mass is felt in the left iliac fossa. USS abdomen suggests a left ovarian tumour. What is the most likely diagnosis?
A) Meningitis
B) Anti-NMDA receptor encephalitis
C) Rabies
D) Japanese encephalitis
E) Mania

A

Anti-NMDA receptor encephalitis is a paraneoplastic syndrome which presents with prominent psychiatric features.

How well did you know this?
1
Not at all
2
3
4
5
Perfectly
81
Q

Which of the following organisms are common causes of peritonitis secondary to peritoneal dialysis?
A) Coag -ve Staph
B) Coag +ve Staph
C) E. coli
D) Gram -ve cocci
E) Group A strep

A

Coagulase-negative Staphylococcus species e.g. Staphylococcus epidermidis and Staphylococcus capitis peritonitis remains a common complication of peritoneal dialysis. S. Aureus would be the next most common organism.

How well did you know this?
1
Not at all
2
3
4
5
Perfectly
82
Q

A 65M presents with haematemesis. He has a past medical history of AF on warfarin. O/E HR 110, BP 94/58, there is dried blood around his mouth. Hb 101, INR 5.4. What is the most appropriate management of his anticoagulation?
A) Stop warfarin for x2 doses
B) Stop warfarin indefinitely
C) Stop warfarin and give IV vit K 3mg
D) Stop warfarin and give PO vit K 3mg
E) Stop warfarin and give IV vit K 5mg and PCC

A

The patient presents with major bleeding (haematemesis, anaemia, tachycardia, hypotension). The management of major bleeding on warfarin is to stop warfarin and give intravenous vitamin K 5mg and prothrombin complex concentrate.

If INR>8 and minor bleeding - stop warfarin and give IV vit K 3mg.

If INR>8 and no bleeding - stop warfarin and give PO vitamin K 3mg.

If INR 5.0-8.0 and no bleeding - stop warfarin for two doses.

How well did you know this?
1
Not at all
2
3
4
5
Perfectly
83
Q

Following a diagnosis of tetanus, what is the most appropriate antibiotic therapy to give with human tetanus immunoglobulin?
A) IV clarithromycin
B) IV benzylpenicillin
C) IV gentamicin
D) IV metronidazole
E) IV ciprofloxacin

A

IV metronidazole

How well did you know this?
1
Not at all
2
3
4
5
Perfectly
84
Q

Which factor would be most associated with a poor prognosis for patients with paracetamol overdose?
A) Hepatic encephalopathy
B) Previous paracetamol overdoses
C) INR 1.3
D) Arterial pH 7.03
E) AST 750 units/L

A

The single most important prognostic indicator in paracetamol overdoses is acidosis. Other indicators of reduced prognosis include a creatinine >300, and increased prothrombin time.

How well did you know this?
1
Not at all
2
3
4
5
Perfectly
85
Q

Which of the following medications is most likely to cause hypoglycaemia?
A) Metformin
B) Iron
C) Prednisolone
D) Propranolol
E) Theophylline

A

Propranolol, salicylates, sodium valproate and severe iron or paracetamol poisoning can cause a modest hypoglycaemia.

86
Q

Which antipsychotic should a patient at risk of T2DM be commenced on?
A) Chlorpromazine
B) Clozapine
C) Haloperidol
D) Olanzapine
E) Risperidone

A

Risperidone is a first-line treatment that carries a low risk of T2DM. Olanzapine causes the biggest risk of T2DM. Typical antipsychotics such as chlorpromazine are not recommended for the initial treatment of schizophrenia and also has a significantly increased risk of developing T2DM.

87
Q

What is the most appropriate initial intervention to prevent recurrence of VT in a patient with AD long QT syndrome and LQT1 genotype?
A) Amiodarone
B) Atenolol
C) Flecainide
D) Permanent pacing
E) Stellate ganglionectomy

A

B-blockade may be enough to prevent VT in patients with long QT syndrome. Permanent pacing is first line in patients with LQT2 and LQT3 genotypes since they are at significant risk of sudden cardiac arrest.

88
Q

A 21F returns from Tanzania with profuse diarrhoea with mucous and blood and significant abdominal pain. She is otherwise well and has a soft but diffusely tender abdomen, particularly on the LHS. Bloods show a mildly raised WCC, raised neutrophils, normal eosinophils and a raised CRP. Stool samples were ++ for blood and flexi sig shows ulcerated colonic mucosa. What is the most likely diagnosis?
A) Amoebiasis
B) Schistosomiasis
C) Strongyloides
D) Eosinophilic gastroenteritis
E) Tropical sprue

A

Gradual onset diarrhoea with stool ++ for blood and no increase in eosinophils along with ulceration on the flexi sig all indicate amoebiasis. Metronidazole is the treatment of choice.

89
Q

What is the MoA of metoclopramide such that it causes side effects of milk leakage from breasts, a loss of libido and amenorrhoea?
A) Stimulates dopamine release by the hypothalamus
B) Direct stimulatory effects on pituitary lactotropes
C) Binds to D2 receptors on pituitary lactotropes
D) Promotes pituitary adenoma growth
E) Inhibits LH secretion by anterior pituitary

A

Metoclopramide is a dopamine antagonist that binds to D2 receptors on pituitary lactotropes, stimulating the release of prolactin by the anterior pituitary, resulting in galactorrhea. Prolactin inhibits GnRH and gonadotropin secretion so oligomenorrhoea and amenorrhoea may occur.

90
Q

Which of the following is true for raloxifene therapy for osteoporosis?
A) Cause PV bleeding at the end of every month
B) Improvement of BMD in spine and hips
C) Increased HDL levels
D) Increased risk of breast cancer
E) Increased risk of clotting

A

Raloxifene is a SERM that exhibits oestrogen agonist effects on bone and lipid metabolism and oestrogen antagonist effects on uterine endometrium and breast tissue. It causes an increase in hot flushes, leg cramps and an increased risk of clots.

It lowers LDL levels, reduces the fracture risk in the spine but has no proven effect in the hips. It has no effect on breast cancer risk.

91
Q

A 21M with Down syndrome presents with worsening SOB over past few months and is now unable to go out with his friends. O/E he is clubbed and cyanosed at rest and has previously been noted to have a harsh systolic murmur at L sternal edge which is now less audible. CXR shows prominent pulmonary vasculature. What is the most likely underlying diagnosis?
A) Aortic stenosis
B) Atrial septal defect
C) HOCM
D) Patent ductus arteriosus
E) Ventricular septal defect

A

AVSDs are the most common cardiac complication seen in Down syndrome with VSDs and ASDs being the next most common. The murmur here is typical of that heard in VSD with the possibility of a significant R-L shunt.

92
Q

PWhat is the best treatment for pityriasis versicolor?
A) 1% hydrocortisone cream
B) Clotrimazole ointment
C) Fluconazole tablets
D) Fusidic acid ointment
E) Ketoconazole tablets

A

Pityriasis versicolor is a yeast infection caused by M. furfur and appears as hypo pigmented, well-demarcated macula’s covered in a fine scale. Treatment is often with topical antifungals such as clotrimazole.

93
Q

A 32F IVDU presents with 1D history of double vision, droopy eyelids and dry throat. She begins to notice limb weakness and respiratory difficulties. O/E she has poorly reactive pupils, ptosis, restricted eye movements, decreased deep tendon reflexes and symmetrical UL and LL weakness, but sensation is normal. CSF is normal, EMG shows increment on repetitive nerve stimulation. What is the most likely diagnosis?
A) Botulism
B) Lambert Eaton syndrome
C) Miller-fisher variant of GBS
D) Myasthenia Gravid
E) Poliomyelitis

A

Clostridium botulinum blocks the release of ACh from the motor nerve terminal. Botulism can be spread by the ingestion of foods contaminated by inadequate sterilisation/wound contamination in IVDU. Early symptoms are blurred vision, dysphagia and dysarthria, impaired pupillary responses to light, reduced tendon weakness and symmetrical limb weakness. Patients may also have evidence of autonomic disturbances such as dry mouth, constipation and urinary retention.

94
Q

Which of the following factors is most likely to precipitate polymorphic VT?
A) Hypoglycaemia
B) Bisoprolol
C) Hypomagnesaemia
D) Dehydration
E) Hyperkalaemia

A

Hypokalemia is the most important cause of ventricular tachycardia (VT) clinically, followed by hypomagnesaemia.

95
Q

Which antibodies are most likely to be present in a patient with dermatomyositis?
A) ANA
B) Anti-centromere
C) Anti-SCL70
D) Anti-Jo-1
E) Anti-Mi-2

A

Dermatomyositis antibodies: ANA most common, anti-Mi-2 most specific.

Anti-Jo-1 antibodies are not commonly seen in dermatomyositis - they are more common in polymyositis where they are seen in a pattern of disease associated with lung involvement, Raynaud’s and fever

96
Q

What is the best treatment option for an acute presentation of cluster headaches?
A) Ibuprofen
B) Acetazolamide + topical pilocarpine
C) Prednisolone
D) S/C sumatriptan
E) Verapamil

A

Acute treatment: subcutaneous sumatriptan + 100% O2

Prophylaxis: verapamil

97
Q

A patient has a fasting blood glucose of 6.3. What is the most likely underlying pathophysiological change?
A) B-cell hyperplasia
B) B-cell atrophy
C) Muscle insulin resistance
D) Hepatic insulin resistance
Adipose tissue insulin resistance

A

There are two main types of IGR:
-Impaired fasting glucose (IFG) (6.1-7.0) - due to hepatic insulin resistance
-Impaired glucose tolerance (IGT) (7.8-11.1) - due to muscle insulin resistance

Patients with IGT are more likely to develop T2DM and cardiovascular disease than patients with IFG

People with IFG should be offered an oral glucose tolerance test to rule out a diagnosis of diabetes.

98
Q

A 35M presents after a night out, having taken an unknown substance. He is known to have a history of depression. O/E: GCS 13/15, pupils are dilated and divergent. He is tachycardic (HR 110/min), BP 124/70mmHg. His ECG shows sinus rhythm, with a lengthened QTc duration (480msec). He is dry to the touch. Which substance is he most likely to have ingested?
A) Cocaine
B) Amphetamine
C) Sertraline
D) Amitriptyline
E) MDMA

A

Although TCAs primarily increase serotonin and noradrenaline in the brain by slowing re-uptake, they also block histamine, cholinergic and alpha 1 receptors. In overdose, the anti-cholinergic effects give dilated pupils, dry skin, confusion, urinary retention and tachycardia. Divergent pupils are a common finding. TCAs are also cardiotoxic by prolonging the QTc interval and widening the QRS complex. This can potentially lead to ventricular arrhythmias. Other effects of TCAs include seizures and a metabolic acidosis.

99
Q

A 27M presents after a syncopal episode. On inspection of the neck veins he has a prominent ‘a’ wave. On auscultation, there is a harsh crescendo-decrescendo systolic murmur that is heard best at the apex and lower left sternal border. Which of the following medications should be avoided in this patient?
A) ACEi
B) B-blockers
C) Disopyramide
D) Nondihydropyridine CCB
E) Warfarin

A

This patient has HOCM. Agents that reduce preload/afterload (such as nitrate, ACE inhibitors, nifedipine-type calcium antagonists) are contraindicated with HOCM due to possible aggravation of the outflow tract obstruction.

Patients with significant symptoms of heart failure and left ventricular outflow tract (LVOT) obstruction benefit from b-blockers and non-dihydropyridine CCBs.

Management of HOCM:
Amiodarone
Beta-blockers or verapamil for symptoms
Cardioverter defibrillator
Dual chamber pacemaker
Endocarditis prophylaxis*

100
Q

Which one of the following statements concerning discoid lupus is correct?
A) Commonly progresses to SLE
B) Causes non-scarring alopecia
C) Characterised by follicular keratin plugs
D) Is rarely photosensitive
E) Typically presents in older males

A

Discoid lupus erythematosus is a benign disorder generally seen in younger females. It very rarely progresses to systemic lupus erythematosus and is characterised by follicular keratin plugs.

Features
-Erythematous, raised rash, sometimes scaly
-May be photosensitive
-More common on face, neck, ears and scalp
-Lesions heal with atrophy, scarring (may cause scarring alopecia), and pigmentation

101
Q

A 25M attends with a 3-month history of numbness in his right hand. OE, you note the loss of sensation to the palmar and dorsal aspect of the 5th digit. Sensation of the forearm is preserved. What is the most likely diagnosis?
A) Axillary nerve neuropathy
B) C8/T1 radiculopathy
C) Carpal tunnel syndrome
D) Cubital tunnel syndrome
E) Radial nerve neuropathy

A

Cubital tunnel syndrome- this is a classical description of ulnar nerve neuropathy. The ulnar nerve supplies sensory innervation to the palmar and dorsal aspects of 1.5 fingers medially. It has a motor component which can result in wasting and paralysis of intrinsic hand muscles (except lateral two lumbricals) and the hypothenar muscles.

C8/T1 radiculopathy can mimic ulnar nerve neuropathy. The preserved sensation of the forearm favours cubital tunnel syndrome as the ulnar nerve does not provide sensation to the medial forearm, which is innervated by the medial antebrachial cutaneous nerve (C8 and T1).

102
Q

An obese diabetic patient needs to uptitrate his diabetic medication. Currently he is only on metformin. He has a history of inflammatory bowel disease, hypertension and a myocardial infarction.
A) Gliclazide
B) Lantus
C) Liraglutide
D) Pioglitazone
E) Semaglutide

A

Semaglutide is a GLP-1 type drug that promotes weight loss and has no contraindications in renal impairment or inflammatory bowel disease.

Pioglitazone promotes fluid retention and hence weight gain and has risks of hypoglycaemia in patients with previous myocardial ischaemia. Sulfonylureas have a tendency to promote weight gain. Insulin is not indicated as a second line treatment. Liraglutide would promote weight loss but is not indicated in inflammatory bowel disease.

103
Q

In a trial of a new cancer drug, the mortality in the control arm was 12.5% and the relative risk of death for patients receiving the new treatment was 0.8. How many patients must be treated with the new drug to save one life?
A) 8
B) 10
C) 25
D) 40
E) 80

A

The number needed to treat (NNT) is the average number of patients needed to be treated to save one life.

NNT = 1 / ARR (absolute risk reduction)
ARR = Control event rate - Experimental event rate

The mortality in the experimental event arm can be found by multiplying the mortality in the control by the relative risk – 12.5% x 0.8 = 10%

ARR = 12.5 - 10 = 2.5%
NNT = 1 / 0.025 = 40

104
Q

What does damage to the recurrent laryngeal nerve cause?
A) Normal vocal cord function
B) Paralysis of cricothyroid muscle on affected side
C) Paralysis of interarytenoid muscle on affected side
D) Paralysis of posterior cricothyroid muscle on affected side
E) SOB when lying flat

A

Recurrent laryngeal nerve injury leads to a hoarse voice due to damage to the posterior cricothyroid muscle which is responsible for opening the vocal cords.

105
Q

What is the second line treatment in a patient with bipolar disorder on lithium with a blood level of 1.5.
A) A trial of imipramine
B) A trial of valproate
C) The addition of haloperidol
D) An increase in lithium
E) A trial of thioridazine

A

It is unwise to increase the lithium level given the level is already at the top of the therapeutic range. The next step would be to use sodium valproate in combination with lithium to treat the condition.

106
Q

A 56M with IE is treated with IV antibiotics. His previously normal ECG now shows an increasing PR interval. His clinical condition has not changed significantly and has been maintaining a low grade fever throughout. What is the most appropriate next course of action?
A) Angiography
B) Blood transfusion
C) Change antibiotics
D) IVF
E) Urgent surgical referral

A

An increasing PR interval suggests an aortic root abscess which has a high mortality. Failure of fever to resolve after 1W also suggests the presence of an intracardiac abscess that requires surgical management.

107
Q

What complication is most likely to be present in rheumatoid arthritis?
A) Telescopic fingers
B) Bakers cysts
C) Mallet finger
D) Onycholysis
E) Heberdens nodes

A

Bakers cysts (popliteal cysts) may occur in RA following persistent effusion into the knee joint.

Telescopic fingers indicate arthritis mutilans.

108
Q

Which result is most characteristic of hypochloraemic alkalosis?
A) Expanded ECF
B) Fall in pCO2
C) Hypokalaemia
D) Low serum bicarbonate level
E) Poor response to ammonium chloride

A

Hypochloraemic alkalosis is associated with severe hypokalaemia.

109
Q

A 42M presents with central crushing chest pain that came on suddenly whilst attending a party. He appears diaphoretic and clammy, describing his pain as 10/10. He has no significant past medical history but discloses he has snorted a significant amount of cocaine whilst at the party. An ECG is performed which shows ST elevation of 4mm in anterior leads. In addition to standard acute coronary syndrome (ACS) treatment, what else is indicated in the management of this patient?
A) IV glucagon
B) IV labetalol
C) IV lidocaine
D) IV lorazepam
E) IV verapamil

A

ACS in the context of cocaine use is often secondary to coronary artery vasospasm, however can also be thrombotic. Standard ACS pathways should be used, with the addition of IV benzodiazepines.

110
Q

What is the first line management if a foetus is found to have a patent ductus arteriosus?
A) Nothing, PDA mostly closes spontaneously
B) The baby will require indomethacin as a neonate
C) The baby will require surgery
D) The baby receives prostaglandin E1 as a neonate
E) The mother should take indomethacin from 36W

A

First-line management of PDA is indomethacin or ibuprofen given to the neonate.

Doing nothing is not the correct answer. Although it can happen, PDA does not close spontaneously in most cases.
Prostaglandin E1 is incorrect. This can be used if PDA is associated with another congenital heart defect (usually cyanotic) amenable to surgery. It is used to keep the duct open until after surgical repair.

111
Q

Which one of the following is not associated with an increased risk of developing torsade de pointes?
A) Romano-Ward syndrome
B) SAH
C) Hypercalcaemia
D) Hypothermia
E) TCA

A

Hypocalcaemia, not hypercalcaemia, causes prolongation of the QT interval and hence may predispose to the development of torsade de pointes

112
Q

A 59M is admitted to hospital following an ischaemic stroke. He presented outside of the thrombolysis window and is treated with aspirin for the first few days. His blood pressure is 130/80 mmHg, fasting glucose is 5.6 mmol/l and fasting cholesterol is 3.9 mmol/l. Following recent NICE guidelines, which of the following medications should he be taking upon discharge (i.e. after 14 days)?
A) Aspirin + statin
B) Aspirin + dipyridamol + statin + ramipril
C) Clopidogrel + statin
D) Aspirin + dipyridamol
E) Aspirin + dipyridamol + statin

A

With regards to secondary prevention, clopidogrel is now recommended by NICE ahead of combination use of aspirin plus modified-release (MR) dipyridamole in people who have had an ischaemic stroke following 14 days of aspirin.

Aspirin 300mg should be given ASAP if a haemorrhagic stroke has been excluded. If the cholesterol is > 3.5 mmol/l patients should be commenced on a statin. Many physicians will delay treatment until after at least 48 hours due to the risk of haemorrhagic transformation

113
Q

A 58M was diagnosed as having type 2 diabetes mellitus (T2DM) 10 years ago and currently takes gliclazide and atorvastatin. He has a past medical history of bladder cancer and is intolerant of metformin. He works as an accountant, is a non-smoker and his BMI is 31 kg/m². His annual bloods show a HbA1c of 62mmol/mol. Which is the best addition to his medication?
A) Pioglitazone
B) Exenatide
C) Acarbose
D) Repaglinide
E) Sitagliptin

A

Pioglitazone is contraindicated by his history of bladder cancer and may contribute to obesity. A DPP-4 inhibitor such as sitagliptin is therefore the best option.

Exenatide generally causes weight loss and is therefore useful in obese diabetics but he does not meet the NICE body mass index criteria of 35 kg/m²

Metformin can be titrated up to aim for a HbA1c of 48 mmol/mol (6.5%) if it rises above 51 mmol/mol (6.8%) but should only add a second drug if the HbA1c rises to 58 mmol/mol (7.5%).

114
Q

When auscultating patient with complete heart block, what is an expected finding?
A) Loud S1
B) Quiet S1
C) Variable S1
D) Loud S2
E) Quiet S2

A

A variable intensity of S1 is consistent with complete heart block.

Loud S1 is caused by mitral stenosis and tricuspid stenosis.

Soft S1 is caused by mitral regurgitation, severe heart failure, left bundle branch block, and 1st-degree heart block.

Loud S2 is caused by systemic hypertension.

Soft S2 is caused by severe aortic stenosis.

115
Q

15M is referred to investigate abnormally tall stature. He has a past medical history of unexplained intellectual disability. O/E, he is 2.08 metres tall, pes cavus and genu valgum are noted. He has long, thin fingers. Cardiovascular and respiratory examinations are normal. Examination of his eyes reveals downward dislocation of the lens. What is the most likely diagnosis?
A) Hyperthyroidism
B) Homocystinuria
C) Marfan syndrome
D) Klinefelter’s syndrome
E) Precocious puberty

A

Homocystinuria is characterized by tall stature, downward lens dislocation (ectopia lentis) and learning difficulties. There is a body habitus that is similar to Marfan’s syndrome but key differentiating characteristics are the downward rather than upward lens dislocation and the presence of learning difficulties.

116
Q

A 37M is anti-CCP negative and HLA-DRB1 positive. He has a family history of joint disease and personal history of intermittent joint pains and swellings. What condition is he most at risk of?
A) Ankylosing spondylitis
B) Rheumatoid arthritis
C) Behcets disease
D) Reactive arthritis
E) Sjogrens syndrome

A

The patient is positive for HLA-DRB1. HLA-DRB1, and its association to HLA-DR4, is a genetic susceptibility locus in rheumatoid arthritis, making this the correct answer.

Behcets disease is associated with HLA-B51. Reactive arthritis, as with ankylosing spondylitis, is related to HLA-B27. Sjogren’s syndrome is related to HLA-DR3.

117
Q

Which of the following antibiotics are associated with a raised bilirubin, raised ALP, raised GGT but normal LFTs?
A) Amiodarone
B) Co-amoxiclav
C) Methotrexate
D) Nitrofurantoin
E) Simvastatin

A

The LFTs demonstrate a cholestatic picture with a raised ALP/GGT and associated hyperbilirubinemia. This is a classical presentation of drug-induced cholestasis. Co-amoxiclav is a well-recognised and common cause of cholestasis.

All the other drugs tend to cause a hepatitis-type picture.

118
Q

Which one of the following is the most significant risk factor for developing lung cancer?
A) Asbestos exposure for most of working life
B) 30 pack year smoking history
C) Cryptogenic fibrosing alveolitis
D) 10 year period of smoking crack 20 years ago
E) 20 year history of recreational cannabis use

A

Smoking increases risk of lung ca by a factor of 10

Other factors
-Asbestos - increases risk of lung ca by a factor of 5
-Arsenic
-Radon
-Nickel
-Chromate
-Aromatic hydrocarbon
-Cryptogenic fibrosing alveolitis

119
Q

A 69M is found to have an elevated serum paraprotein level of 35g/L. Bone marrow aspirate reveals 32% monoclonal plasma cell infiltrate. He has no evidence of anaemia, renal impairment, hypercalcaemia or lytic lesions. What is the next step in management?
A) Observe and monitor
B) Arrange for autologous stem cell transplant
C) Commence thalidomide
D) Commence dexamethasone
E) Commence combined therapy with prednisolone and thalidomide/bortezomib

A

Because the patient is asymptomatic but has the criteria for multiple myeloma, the underlying diagnosis of this stem is smoldering multiple myeloma. The treatment of smoldering multiple myeloma is typically to watch and wait

120
Q

What is the pharmacological action of sildenafil?
A) a-glycosidase blocker
B) COX-II antagonist
C) H+/K+ ATPase antagonist
D) Phosphodiesterase V antagonist
E) Topoisomerase I antagonist

A

Sildenafil is a PO drug approved for the treatment of erectile dysfunction. It selectively inhibits type-V phosphodiesterase that is found in human penile erectile tissue. It may interact with nitrates to cause hypotension and syncope; adverse effects include flushing, headache, dyspepsia and nasal congestion.

121
Q

A 51M presents with flushing, wheeze and weight loss. He is found to have hepatomegaly, elevated liver transaminases and imaging shows multiple hepatic lesions. Urinary 5HIAA are found to be elevated. What other symptom would you expect to be found in this patient?
A) Diarrhoea
B) Abdominal pain
C) Hypertension
D) Skin rash
E) Headache

A

Carcinoid syndrome is a paraneoplastic syndrome that occurs as a result of carcinoid tumours. It leads to symptoms including flushing and diarrhoea and less frequently heart failure and bronchoconstriction.

122
Q

A 25F is treated for a PE. She is started on unfractionated IV heparin and then switched to warfarin, but is slow to reach therapeutic anticoagulation and heparin is therefore continued. 8 days into treatment she is requiring increasing doses of heparin to maintain a therapeutic APTT. On day 10 she has a pale, pulseless left foot. Which of the following results support the clinical diagnosis?
A) A low platelet count 50x10(9)
B) Low factor VII assay
C) Low protein C level (43%)
D) APTT ratio 1.7
E) Prolonged INR

A

This woman has had an arterial embolic event despite heparin anticoagulation. Therefore this is suspicious for HIT and a low platelet count would be supportive of this.

123
Q

Which of the following is recommended for post-exposure prophylaxis for people exposed to needle-stick injuries?
A) Zidovudine + lamivudine + indinavir for 28 days
B) Truvada (tenofovir + emtricitabine) + raltegravir for 28 days
C) Zidovudine + lamivudine + tenofovir for 28 days
D) Truvada (tenofovir + emtricitabine) for 28 days
E) Zidovudine + stavudine + nevirapine for 28 days

A

28 days of Truvada + raltegravir should be used for all PEP.

124
Q

What is the best treatment option for a patient with blunt ocular trauma presenting with: eye pain, hazy cornea, hyphaemia and a firm globe on palpation?
A) Examination under anaesthesia
B) Topical B-blockers
C) IV carbonic anhydrase inhibitor
D) Topical anticholinergics
E) Anterior chamber paracentesis

A

IV carbonic anhydrase inhibitor will quickly lower intraocular pressure, before optic nerve damage or even retinal artery compression and occlusion occur.

Hyphaemia occurs when blood from iris vessels fills the anterior chamber and can be quantified in terms of its % filling. If large, they are at risk of causing a raised IOP as blood cells block the trabecular meshwork and impairs the drainage of aqueous humour. One sign of raised IOP is corneal oedema and hence corneal haziness.

125
Q

Which of the following medications would be most appropriate to prescribe to treat hypertension in a women wanting to get pregnant?
A) Ramipril
B) Nifedipine
C) Candesartan
D) Hydrochlorothiazide
E) Bendrofluazide

A

Nifedipine should be prescribed to a patient over the age of 30 with uncontrolled hypertension. CCBs however may delay the onset of labour.

ACEi should not be used in pregnant women due to the adverse risks to the foetus.

126
Q

A student returns from a backpacking holiday with abdominal pain, diarrhoea and fevers. O/E he has a fever (38.5) and diffuse abdominal pain. Which of the following is the most likely causative organism?
A) Clostridium difficile
B) Plasmodium falciparum
C) Norovirus
D) Salmonella spp.
E) Vibrio cholerae

A

The presence of bloody diarrhoea suggests a GI infection by an enteroinvasive organism. The most common are Salmonella spp, Shigella spp. and Enterohaemorrhagic E Coli. Entomoeba histolytica can also cause bloody diarrhoea but these symptoms progress more slowly.

127
Q

Persistent hyperkalaemia is associated with which of the following?
A) Gitelmans syndrome
B) Liddle’s syndrome
C) Renal tubular acidosis type 1
D) Renal tubular acidosis type 2
E) Renal tubular acidosis type 4

A

Renal tubular acidosis 4 results from a deficiency of mineralocorticoids - e.g. through medications (NSAIDs, ACEi), Addisons disease or congenital enzyme defects. The aldosterone deficiency means there is less sodium resorption and hence less potassium excretion, leading to hyperkalaemia. Acidosis is uncommon in RTA4.

RTA1 is seen in many renal disorders - e.g. AI disease, rejection of transplanted kidney, chronic pyelonephritis or medications such as lithium. It leads to the reduced excretion of hydrogen ions, causing a metabolic acidosis and hence hypokalaemia.

RTA2 is much less common than RTA1 and occurs due to a defect in bicarbonate resorption leading to metabolic acidosis and hence hypokalaemia.

128
Q

Which of the following are most likely to cause upper lobe fibrosis on CXR?
A) Ankylosing spondylitis
B) Idiopathic pulmonary fibrosis
C) Rheumatoid arthritis
D) Scleroderma
E) SLE

A

Causes of upper lobe fibrosis on CXR:
-Ankylosing sponylitis
-Sarcoidosis
-Tb
-EAA
-Silicosis
-ABPA
-Post-radiotherapy

129
Q

A man with AF is due for an elective DC cardioversion but his procedure is postponed. Which of the following could be the cause of delaying his procedure?
A) He had discontinued digoxin for the past 2 days
B) He was taking amiodarone
C) His INR 2 weeks ago was 1.6
D) His K+ was 4.6
E) He had an episode of angina 2 weeks ago

A

The INR should be >2 on the day of the procedure and the preceding 3-4 weeks due to risk of dislodging a clot.

130
Q

A 34-year-old man with a history of polyarthralgia, back pain and diarrhoea is found to have a 3 cm red lesion on his shin which is starting to ulcerate. What is the most likely diagnosis?
A) Systemic shigella infection
B) Syphilis
C) Metastatic colon cancer
D) Erythema nodosum
E) Pyoderma gangrenosum

A

This patient is likely to have ulcerative colitis, which has a known association with large-joint arthritis, sacroilitis and pyoderma gangrenosum

131
Q

Each one of the following physiological changes occur during exercise, except:
A) Increased myocardial contractility
B) 50% increase in stroke volume
C) 3-fold increase in HR
D) Rise in diastolic BP
E) Venous constriction

A

Blood pressure: systolic increases, diastolic decreases which leads to an increased pulse pressure - in healthy young people the increase in MABP is only slight.

132
Q

An 81-year-old man attends the emergency department with acute knee pain. He has no other symptoms.
X-ray right knee shows normal joint space. Prominent calcification of the menisci and articular cartilage
Synovial fluid MC&S: White blood cells: 1700/mm³. No growth at 48 hours.
A) Gout
B) OA
C) Pseudogout
D) Reactive arthritis
E) Septic arthritis

A

The key clue here is chondrocalcinosis, the finding of calcification of the articular cartilage. Although not specific, this is suggestive of pseudogout. In the knee, this often manifests as calcification of the menisci.

133
Q

A 45-year-old man is admitted due to haematemesis. He reports drinking 120 units of alcohol a week. When is the peak incidence of seizures following alcohol withdrawal?
A) 2 hours
B) 6 hours
C) 12 hours
D) 36 hours
E) 72 hours

A

-Alcohol withdrawal
symptoms: 6-12 hours
-Seizures: 36 hours
-Delirium tremens: 72 hours

134
Q

What is the best management for a patient with ITP with minor bleeding and a platelet count of 26x10(9)?
A) Observation only
B) IV corticosteroids
C) IVIG
D) Oral prednisolone
E) Platelet transfusion

A

Appropriate first-line treatment would be oral prednisolone.

Corticosteroids help suppress the immune-mediated destruction of platelets in severe/critical bleeding. IVIG is a second-line therapy for minor bleeding, or first-line (along with other agents) for severe bleeding. Observation only is reserved for asymptomatic adults with a platelet count >=30 * 109/L. Platelet transfusions are reserved for those with life- or organ-threatening bleeding and may be used to control active bleeding for surgery; however, they are ineffective as a treatment for ITP due to continued immune-mediated platelet destruction.

135
Q

Which of the following medications is most likely to contribute to drug-induced cholestasis?
A) Methotrexate
B) COCP
C) Nitrofurantoin
D) Phenytoin
E) Sodium valproate

A

The following drugs tend to cause a hepatocellular picture:
-paracetamol
-sodium valproate, phenytoin
-MAOIs
-halothane
-anti-tuberculosis: isoniazid, rifampicin, -pyrazinamide
-statins
-alcohol
-amiodarone
-methyldopa
-nitrofurantoin

The following drugs tend to cause cholestasis (+/- hepatitis):
-COCP
-antibiotics: flucloxacillin, co-amoxiclav, erythromycin
-anabolic steroids, testosterones
-phenothiazines: chlorpromazine, -prochlorperazine
-sulphonylureas
-fibrates

Liver cirrhosis:
-methotrexate
-methyldopa
-amiodarone

136
Q

A 17-year-old man attends the local sexual health clinic. Over the last 3 months he has developed a large, keratinised genital wart on the shaft of his penis. What is the most appropriate initial management?
A) Topical podophyllum
B) Topical aciclovir
C) Electrocautery
D) Topical salicylic acid
E) Cryotherapy

A

Genital warts are caused by human papillomavirus HPV, especially types 6 & 11. It is well established that HPV (types 16,18 & 33) predisposes to cervical cancer. Topical podophyllum or cryotherapy are commonly used as first-line treatments depending on the location and type of lesion:
-multiple, non-keratinised warts are best treated with topical agents
-solitary, keratinised warts respond better to cryotherapy

Imiquimod is a topical cream that is generally used second line.

137
Q

Which one of the following would shift the oxygen dissociation curve to the right?
A) HbF
B) Increased 2,3-DPG
C) Methaemaglobin
D) Alkalosis
E) High pCO2

A

The oxygen dissociation curve describes the relationship between the percentage of saturated haemoglobin and partial pressure of oxygen in the blood.

-Shifts to left = for given oxygen pressure, there is increased saturation of Hb with oxygen i.e. decreased oxygen delivery to tissues
-Shifts to right = for given oxygen pressure, there is reduced saturation of Hb with oxygen i.e. enhanced oxygen delivery to tissues

Shifts to Left = Lower oxygen delivery
-HbF, methaemoglobin, carboxyhaemoglobin
-Low [H+] (alkali)
-Low pCO2
-Low 2,3-DPG
-Low temperature Raised [H+] (acidic)

Shifts to Right = Raised oxygen delivery
-Raised pCO2
-Raised 2,3-DPG*
-Raised temperature

138
Q

Which of the following antibodies is most specific for dermatomyositis?
A) Anti-Jo1
B) ANA
C) Anti-scl70
D) Anti-centromere
E) Anti-Mi2

A

Dermatomyositis antibodies: ANA most common, anti-Mi-2 most specific

Ix
-elevated creatine kinase
-EMG
-muscle biopsy
-ANA positive in 60%
-anti-Mi-2 antibodies are highly specific for dermatomyositis, but are only seen in around 25% of patients

-anti-Jo-1 antibodies are more common in polymyositis.

Management
-prednisolone

139
Q

A 49-year-old homeless male presents with a reduced GCS of 14/15. His pupils are equal but poorly reactive to light and he is complaining of poor eyesight. An ABG shows a metabolic acidosis. Which substance is he most likely to have ingested?
A) Aspirin
B) Alcohol
C) Ethanol
D) Amitriptyline
E) Ethylene glycol

A

All of the answer options can cause a metabolic acidosis. As this man is homeless and there is no further information given, the most likely causes for his presentation are alcohol, methanol and ethylene glycol (anti-freeze). Reduced vision and poorly reactive pupils are common complications of methanol poisoning. A metabolite of methanol, formic acid, accumulates in the optic nerve causing visual disturbance and eventually blindness.

140
Q

A 19F presented with a gradual development of B/L hearing loss accompanied by nausea and ringing in the ears but denied otalgia or otorrhoea. She reported a family history of hearing problems but was unsure what the exact condition was. Otoscopic examination demonstrated a reddish blush visible on the cochlear promontory beyond an intact tympanic membrane. What is the diagnosis in this case?
A) Adhesive otitis media
B) Cerumen impaction
C) Menieres disease
D) Otosclerosis
E) Typanosclerosis

A

Otosclerosis is characterised by conductive hearing loss, tinnitus and positive family history. It is a form of hearing loss that occurs due to abnormal bone growth in your middle ear (due to fixation of the stapes at the oval window). In 10% of cases of otosclerosis, there is a redness of the promontory of the cochlea seen through the tympanic membrane due to prominent vascularity known as the Schwartz sign, which is seen in this case.

Meniere’s disease produces a classic triad of symptoms (episodic vertigo, tinnitus, and hearing loss), likely caused by endolymphatic hydrops of the labyrinthine system of the inner ear. The absence of recurrent rotatory vertigo makes this diagnosis unlikely.

141
Q

A 59M presents with severe crushing chest pain going down his left arm, sweating and syncope. He smokes 20 cigarettes/day. O/E his BP is 90/60 and has B/L basal crackles. His ECG shows T wave inversion in V5 and V6. Which of the following is the most likely finding on angiography?
A) Complete occlusion of the LAD
B) Critical stenosis of the left circumflex
C) Complete occlusion of the right coronary
D) Critical stenosis of the right coronary
E) Critical stenosis of the LAD

A

The left circumflex artery supplies the lateral and posterior left ventricular walls. It is likely to be critical stenosis rather than complete occlusion due to the presence of T-wave inversion rather than ST-elevation.

142
Q

Which of the following would exacerbate digoxin poisoning?
A) Hypokalaemia
B) Hypocalcaemia
C) Hypermagnesaemia
D) Hyponatramia
E) Hypophosphataemia

A

Potassium depletion enhances the binding of digoxin to the Na+/K+ pump which increases the extent of inhibition of sodium transport, which in turn, causes digoxin toxicity.

143
Q

A 49F is currently under section on a psychiatric ward for major depressive disorder. She has repeatedly stated that she would be better off dead and has been refusing food and drink. Her most recent bloods show an AKI and she has been started on IVF in A&E. What would be the best way to manage her depression?
A) ECT
B) Fluoxetine via NGT
C) IM antipsychotics
D) MAOI via NGT
E) Sedation and further IVF

A

A course of ECT is sometimes used as treatment for severe depression if there is a risk to a persons life and urgent treatment is required, or for moderate or severe depression when all other treatments have not helped. Indications for ECT in depression include:
-Depressive stupor or catatonia
-Psychotic depression
-Refusal of food or drink
-When the patient is dangerously suicidal
-Situations where a delay in the effects of the anti-depressants will result in serious health risk

144
Q

What is the best treatment for eosinophilic pneumonia?
A) Clarithromycin
B) Inhaled B-agonist
C) LTRA
D) Nebulised B-agonist
E) Systemic steroids

A

Eosinophilic pneumonia often needs treatment with corticosteroid medications - treatment may need to be >6M in patients with chronic forms of the disease. It is also vital to remove any possible causal factors.

145
Q

Which of the following medications would be most appropriate to commence in a hyperthyroid woman who is 12 weeks pregnant?
A) Carbimazole and thyroxine
B) Radioactive iodine
C) Propranolol
D) Propylthiouracil
E) Surgical resection

A

Propylthiouracil is the intervention of choice in pregnant women and should be used to maintain thyroid levels in the upper third of the normal range. It should not be sued late in pregnancy due to the risk of maternal liver toxicity.

146
Q

What is the most appropriate treatment for a patient with Paget’s disease presenting with bone pain, stiffness and a raised ALP to reduce their symptoms?
A) PO naproxen
B) PO risedronate
C) IV zolendronate
D) PO calcium and vitamin D
E) S/C denosumab

A

When there are symptoms of bone pain and stiffness, coupled with a raised ALP, bisphosphonates are the treatment of choice. If there is a history of gastric issues then IV would be indicated over PO.

147
Q

A 22 year old, otherwise well, presents with widespread rash and fever, thought to be chickenpox. He has a scattered, vesicular rash worst on his torso. His obs are stable. What is the best course of treatment?
A) IV aciclovir
B) Antibiotics to cover for superimposed infection
C) PO aciclovir
D) Paracetamol, PO fluids
E) VZIG

A

Adults presenting with chickenpox within 72 hours of the rash should be treated with PO aciclovir IF: they are immunocompromised/rash on non-truncal areas/moderate or severe pain and moderate or severe rash as these patients are at an increased risk of complications.

148
Q

A 49F presents with worsening TB symptoms and progression of her consolidation on CXR despite being treated with rifampacin, isoniazid and and pyrazinamide. What is the most appropriate next treatment step?
A) Add a fourth drug
B) Add a fourth and fifth drug
C) Continue the three-drug regimen
D) Switch to a macrolide-rifampicin based regime
E) Switch to macrolide, rifampicin and linezolid based regime

A

Progression of TB during first line treatment indicates the possibility of MDR TB which should mean that this patient should be escalated to a 5-drug regime therapy, the specific choices depend on the drugs the strain is resistant to.

149
Q

A 75M with isolated systolic hypertension, who also has urinary incontinence, gout and asthma, attends the OPD for a review. His BP is 190/86. Which of the following drugs would be most appropriate for this patient?
A) Amlodipine
B) Atenolol
C) Bendrofluazide
D) Doxazosin
E) Valsartan

A

Following the HTN treatment ladder, this patient is over the age of 55 and does not have diabetes - hence should be started on a CCB first-line. Amlodipine is the drug of choice for isolated systolic hypertension in the elderly.

150
Q

Which of the following is the most common complication of ciclosporin use?
A) Hypoglycaemia
B) Night sweats
C) Pancreatitis
D) Thrombocytopaenia
E) Tremor

A

Tremor is a common adverse effect of calcineurin inhibitors

150
Q

Which of the following medications should be avoided in a patient with epilepsy taking sodium valproate?
A) Co-amoxiclav
B) Nitrofurantoin
C) Cefixime
D) Trimethoprim
E) Ciprofloxacin

A

Ciprofloxacin lowers the seizure threshold. Quinolones should be used with caution in patients with a history of epilepsy, or conditions that predispose to seizures

151
Q

Lateral medullary syndrome is caused by occlusion of which one of the following blood vessels?
A) Anterior inferior cerebellar artery
B) Posterior cerebral artery
C) Lateral sinus thrombosis
D) Middle cerebral artery
E) Posterior inferior cerebellar artery

A

Lateral medullary syndrome, also known as Wallenberg’s syndrome, occurs following occlusion of the posterior inferior cerebellar artery. It presents with: cerebellar signs, contralateral sensory loss & ipsilateral Horner’s

152
Q

Which of the following is incorrect regarding TCA overdose?
A) Anticholinergic features are prominent early on
B) Metabolic acidosis is a common complication
C) ECG changes include prolongation of QT interval
D) Dialysis is indicated in severe cases
E) QRS interval >160ms is associated with ventricular arrhythmias

A

Dialysis is not indicated in severe toxicity.

Early features relate to anticholinergic properties: dry mouth, dilated pupils, agitation, sinus tachycardia, blurred vision.

Features of severe poisoning include:
-arrhythmias
-seizures
-metabolic acidosis
-coma

ECG changes include:
-sinus tachycardia
-widening of QRS
-prolongation of QT interval

Widening of QRS > 100ms is associated with an increased risk of seizures whilst QRS > 160ms is associated with ventricular arrhythmias

Management includes the use of IV sodium bicarbonate.

153
Q

What is the first-line management for flexural psoriasis?
A) Coal tar preparations
B) Tacrolimus ointment
C) Topical steroids
D) Topical steroids + Vitamin D analogues
E) Vitamin D analogues

A

Topical steroids is the first-line NICE recommendation for treatment of flexural psoriasis. Mild to moderate topical steroids are prescribed for no longer than 2 weeks at a time.

NICE recommend a step-wise approach for chronic plaque psoriasis:
-regular emollients may help to reduce scale loss and reduce pruritus
-first-line: A potent corticosteroid applied once daily plus vitamin D analogue applied once daily for up to 4 weeks as initial treatment
-second-line: if no improvement after 8 weeks then offer a vitamin D analogue twice daily
-third-line: if no improvement after 8-12 weeks then offer either:
a potent corticosteroid applied twice daily for up to 4 weeks, or
a coal tar preparation applied once or twice daily
-short-acting dithranol can also be used

154
Q

Which of the following, if present, would suggest a worse prognosis in a patient presenting with paracetamol overdose?
A) Acute alcohol intake
B) Pre-existing carbamazepine use
C) Concurrent co-amoxiclav
D) Obesity
E) Previous paracetamol overdose

A

Carbamazepine is an inducer of CYP enzymes and hence acts to increase the conversion of paracetamol to its toxic metabolite, NAPQI. This has been demonstrated to worsen the prognosis of patients who take a paracetamol overdose.

The following groups are at an increased risk of developing hepatotoxicity following a paracetamol overdose:
-patients taking liver enzyme-inducing drugs (rifampicin, phenytoin, carbamazepine, chronic alcohol excess, St John’s Wort)
-malnourished patients (e.g. anorexia nervosa) or patients who have not eaten for a few days

155
Q

A 66M attends his GP surgery for a routine health check. His blood pressure is noted to be 152/86 mmHg. He is asymptomatic and examination is unremarkable. He has no significant past medical history. The blood pressure is repeated and is recorded as 155/82 mmHg. What should be done next?
A) Ambulatory BP monitoring
B) Commence ramipril
C) Commence amlodipine
D) Recheck in clinic in one week
E) Recheck later today with nurse

A

Unless the blood pressure is extremely high and requires immediate treatment, NICE now recommends ambulatory or home blood pressure monitoring (ABPM/HBPM), as this provides more reliable readings and avoid white-coat hypertension.

156
Q

A 24M is treated with LMWH following a DVT of his right leg. He is switched after three days to warfarin but then develops necrotic skin lesions on his lower limbs and forearms. Which one of the following conditions is characteristically associated with this complication?
A) Protein S deficiency
B) Antiphospholipid syndrome
C) Antithrombin III deficiency
D) Activated protein C resistance
E) Protein C deficiency

A

Protein C deficiency is an autosomal codominant condition which causes an increased risk of thrombosis. It increases the risk of venous thromboembolism and of skin necrosis following the commencement of warfarin

157
Q

A 34M from Zimbabwe is admitted with abdominal pain. An abdominal x-ray reveals urinary bladder calcification. What is the most likely cause?
A) Schistosoma mansoni
B) Sarcoidosis
C) Leishmaniasis
D) TB
E) Schistosoma haematobium

A

Schistosomiasis is the most common cause of bladder calcification worldwide. Schistosoma haematobium causes haematuria and the eggs can appear as calcification in the bladder. This typically presents as a ‘swimmer’s itch’ in patients who have recently returned from Africa. Schistosoma haematobium is a risk factor for squamous cell bladder cancer.

158
Q

Which one of the following types of glomerulonephritis is most characteristically associated with streptococcal infection in children?
A) FSGS
B) Membranous
C) Diffuse proliferative
D) Mesangiocapillary
E) Rapidly progressive

A

Diffuse proliferative glomerulonephritis, causes:
-post-streptococcal
-SLE

159
Q

A 22M presents with 4 episodes of non-bloody diarrhoea in 24-hours on a background of UC. Observations are within normal limits. He is started on rectal aminosalicylates however there is no improvement. Colonoscopy shows diffuse superficial ulceration from the rectum to the hepatic flexure. What treatment is indicated?
A) IV ciclosporin
B) IV steroids
C) Methotrexate
D) Oral aminosalicylates
E) Rectal mesalazine

A

The patient has a mild flare-up of UC with demonstrable disease extending past the left-sided colon, oral aminosalicylates are the agent of choice as enemas only reach the splenic flexure.

Intravenous steroids are usually given first-line for severe colitis. Features of severe disease include >6 bowel movements per day with systemic upset, visible blood, pyrexia, tachycardia (>90 beats per minute), anaemia, and an ESR >30mm/hour.

160
Q

A 36F presents with a one-hour history of a severe headache. You review her within a few minutes of arrival in the department. She describes a sudden onset headache and feels it is the worst headache she has ever had in her life. She has felt nauseated since it started and vomited once and also describes photophobia. You suspect a subarachnoid haemorrhage and a CT brain is performed one hour after your assessment. The CT shows no acute intracranial pathology.
What is the most appropriate next step in the management of this patient?
A) CT angiogram
B) Consider an alternative diagnosis
C) LP
D) Digital subtraction angiography
E) MRA

A

The NICE guidelines suggest that if a patient has a normal CT brain within 6 hours of the onset of the headache then SAH is unlikely and you should consider an alternative diagnosis.

If the CT is performed greater than 6 hours after the onset of the headache and shows no evidence of SAH then you should consider a lumbar puncture, which should be performed at least 12 hours after onset.

If the CT shows a subarachnoid haemorrhage, the next investigation is usually a CT angiogram.

161
Q

A 23F experiences nausea, vomiting and abdominal pain 4 hours after eating salad and a hamburger. Watery diarrhoea begins a few hours later. O/E she appears dehydrated, bowel sounds are present and has tenderness on deep palpation. What is the most likely causative organism?
A) Salmonella eneteridis
B) Listeria monocytogenes
C) Staphylococcus aureus
D) Clostridium perfringens
E) Yersinia enterocolitica

A

S. Aureus food poisoning leads to profuse vomiting that occurs 2-4 hours after eating food (often pre-made salads, meat or dairy products) that is contaminated by an enterotoxin. Diarrhoea occurs a few hours later that is profuse and watery.

162
Q

A 70M presents acutely confused with a plasma sodium of 108mmol/L. His creation is 106 mol/L and urea 5.0mmol/L. Which of the following findings would be most suggestive of a diagnosis of SIADH?
A) Plasma albumin concentration of 28g/l
B) Plasma osmolality of 230 mosmol/kg
C) Plasma urea concentration 2.5mmol/l
D) Spot urine osmolality 510 mosmol/kg
E) Urine sodium concentration 15 mmol/L

A

The finding of a urine osmolality higher than plasma implies defective water excretion and is characteristic of SIADH.

The plasma osmolality is likely to be low (2Na + U + G) and the low plasma osmolality could just be due to fluid overload rather than SIADH.

163
Q

For a patient presenting with symptoms of MS, apart from MRI scanning, which of the following investigations may be helpful in diagnosis?
A) EEG
B) EMG
C) Nerve conduction studies
D) Polysomnography
E) Visual evoked potentials

A

Although current diagnostic criteria for MS rely on MRI scanning, visual evoked potentials (VEP) and LP may be performed in addition. VEPs are averaged evoked potentials evoked by visual stimuli and detected by scalp electrodes placed over the occiput. It is sensitive for detecting demyelination of the optic nerve and central visual pathways.

Nerve conduction studies are not helpful in the diagnosis of MS as it is a purely central nerve disorder.

164
Q

Which of the following is a potential complication of hypothyroidism?
A) Cardiomegaly
B) Glove and stocking peripheral neuropathy
C) Gynaecomastia
D) Optic neuropathy
E) Pretibial myxoedema

A

Patients with severe hypothyroidism are at risk of developing pericardial effusions, cardiomegaly and heart failure.

Hypothyroidism may cause nerve entrapment syndrome but do not cause length-dependent neuropathy. Gynaecomastia, optic neuropathy and pre-tibial myxoedema are all complications of hyperthyroidism.

165
Q

Which of the following medications interact with warfarin to cause a raised INR?
A) Carbamazepine
B) Ciprofloxacin
C) Phenobarbital
D) Rifampicin
E) Primidone

A

Chloramphenicol, ciprofloxacin, erythromycin, metronidazole and omeprazole are all hepatic enzyme inhibitors.

Phenobarbital, rifampicin and carbamazepine are all hepatic enzyme inducers.

166
Q

Which of the following is a good first choice agent in treating primary generalised epilepsy in a 25M?
A) Carbamazepine
B) Phenytoin
C) Topiramate
D) Phenobarbitol
E) Valproate

A

The first choice treatments for primary generalised epilepsy are lamotrigine and valproate. In women of child-bearing age, valproate is avoided.

167
Q

A 78F presents with increasing SOB, a productive cough, fevers and red, jelly-like sputum. She has a background of previous alcohol dependence, T2DM and a recent ischaemic stroke. She is currently risk feeding and awaiting SALT follow up. CXR demonstrates RUZ consolidation. What is the most likely cause of the pneumonia?
A) Klebsiella pneumoniae
B) Haemophilus influenzae
C) Mycoplasma
D) Staphylococcus aureus
E) Streptococcus pneumoniae

A

Klebsiella pneumonia is more common in diabetics and patients with a history of alcohol excess. It is also frequently caused by aspiration. Here, the recent stroke has caused problematic dysphagia. Klebsiella commonly affects the upper lobes of the lungs.

168
Q

Which investigation is needed in the investigation of a patient presenting with CLL symptoms and smudge cells on his blood film?
A) Coombs test
B) Cytogenetic testing
C) Immunoglobulin assay
D) Immunophenotyping
E) Lymph node biopsy

A

The investigation of CLL includes a full blood count, blood film which may show the presence of smudge or smear cells and a bone marrow aspiration to demonstrate infiltration with lymphocytes. Immunophenotyping is an important investigation for diagnosing CLL and has a role in predicting prognosis. Most cases can be identified using a panel of antibodies specific for CD5, CD19, CD20 and CD23

169
Q

A 74-year-old woman develops tense, itchy blisters on her inner thighs and upper arms. Given the likely diagnosis, what will immunofluorescence of the skin biopsy demonstrate?
A) Loss of fibrinogen at BM
B) Granular IgG at BM
C) IgM crystallisation at dermal junctions
D) Linear IgA deposit at dermoepidermal junction
E) IgG and C3 at the dermoepidermal junction

A

Bullous pemphigoid is an AI condition causing sub-epidermal blistering of the skin. This is secondary to the development of antibodies against hemidesmosomal proteins BP180 and BP230.

Bullous pemphigoid is more common in elderly patients. Features include
-itchy, tense blisters typically around flexures
-the blisters usually heal without scarring
-there is stereotypically no mucosal involvement (i.e. the mouth is spared)

170
Q

Which of the following is a RF for developing MALT lymphoma?
A) Alcoholism
B) Current smoker
C) GORD
D) Hashimoto’s thyroiditis
E) T2DM

A

Research shows Hashimoto’s thyroiditis is amongst the autoimmune diseases found to be associated with the development of MALT lymphomas. Current theories suggest that lymphoma develops in response to areas of localised autoimmune stimulation.

171
Q

Which of the following organisms is the most common causative organism of an infective exacerbation of COPD?
A) Streptococcus pneumoniae
B) Klebsiella pneumonia
C) Haemophilus influenza
D) Moraxella catarrhalis
E) Mycoplasma pneumoniae

A

The most common organism causing infective exacerbations of COPD is Haemophilus influenzae. Other bacterial causes include Streptococcus pneumoniae and Moraxella catarrhalis. Respiratory viruses account for around 30% of exacerbations, with the human rhinovirus being the most common.

172
Q

Which of the following is an early feature of rheumatoid arthritis?
A) Joint subluxation
B) Juxta-articular osteopaenia
C) Peri-articular erosions
D) Subchondral cysts
E) Subchondral sclerosis

A

Juxta-articular osteopenia, along with loss of joint spae and soft-tissue swelling, is a common early X-ray feature of rheumatoid arthritis whilst joint subluxation and peri-articular erosions often occur in the context of progressive disease.

173
Q

Which of the following investigations is the most suitable to detect chronic pancreatitis?
A) ERCP
B) Endoscopic USS
C) US Abdomen
D) CT Abdomen
E) Endoscopy with D2 biopsy

A

CT is the most sensitive method to detect the characteristic pancreatic calcification which is associated with the condition. Functional tests: faecal elastase may be used to assess exocrine function if imaging inconclusive

173
Q

Which of the following pharmacological treatment is the most appropriate for treatment of bulimia?
A) Low dose citalopram
B) Low dose fluoxetine
C) Low dose amitriptyline
D) High dose amitriptyline
E) High dose fluoxetine

A

If bulimia-nervosa-focused guided self-help is unacceptable, contraindicated, or ineffective after 4 weeks of treatment, NICE recommend individual eating-disorder-focused cognitive behavioural therapy (CBT-ED). Children should be offered bulimia-nervosa-focused family therapy (FT-BN). Pharmacological treatments have a limited role - a trial of high-dose fluoxetine is currently licensed for bulimia but long-term data is lacking.

174
Q

What aspect of the genetics of a disease gives rise to a variation in symptoms experienced by those who are diagnosed with Marfans syndrome?
A) Anticipation
B) Expressivity
C) Inheritance
D) Methylation
E) Penetrance

A

Expressivity refers to the extent to which a particular genotype is expressed in the phenotype of an individual. Marfan’s disease has a very varied expressivity, meaning different people affected by it can be affected very differently.

Anticipation is the phenomenon in which the age of onset of a condition lowers with each successive generation. Classically this is seen in trinucleotide repeat disorders such as Huntington’s disease (CAG expansion), myotonic dystrophy (CTG), and fragile X-syndrome (CGG). Methylation is a process by which genes can be silenced by the addition of methyl groups. Penetrance refers to the proportion of the population who have a particular mutation also having the associated disease. An example would be BRCA-1, a mutation of which causes breast cancer in 80% of those affected, giving it a penetrance of 80%.

175
Q

Which of the following cytokines is involved in the pathophysiology of sepsis?
A) Histamine
B) IL1
C) IL4
D) IL5
E) IL8

A

IL-1, IL-6, and TNF-a are involved in the complex cytokine cascade during sepsis. Sepsis triggers the release of IL-1 causing vasodilation → hypotension.

176
Q

A 56F is brought in to ED by a concerned neighbour. Beside the patient are empty blister packets of co-codamol 30/500, indicating that she may have taken up to 50 tablets. She is confused with a GCS of 14/15 and is unable to say when she took the tablets. What is the most appropriate initial management?
A) Start NAC immediately
B) Immediate referral for haemodialysis
C) Give naloxone
D) Start NAC 4hrs after presentation if levels are elevated
E) Observe

A

She may have consumed 25g of paracetamol which is a life-threatening overdose. N-acetyl cysteine needs to be commenced immediately.

NAC should be given immediately if there is a staggered overdose, uncertainty regarding the timing of the overdose or if a patient presents within 8-24hrs after an overdose of more than 150mg/kg.

177
Q

The most common type of inherited colorectal cancer:
A) HNPCC
B) FAP
C) Peutz-Jeghers
D) Fanconi
E) Li-Fraumeni

A

HNPCC

178
Q

A 35-year-old male presents with a four-day history of painless visible haematuria, mostly occurring in the morning. His PMHx indicates a L sided PE. Which of the following investigations is the gold standard for the likely underlying diagnosis?
A) Flow cytometry
B) Guthrie’s test
C) Hams test
D) Sucrose lysis test
E) Urine culture

A

Patients with haematuria and unexplained or unusual sites of thrombosis, or with a raised LDH (indicating haemolysis), abnormal cell counts or aplastic anaemia, should be screened for PNH. PNH is a rare acquired disorder characterised by intravascular haemolysis due to deficiency of decay-accelerating factor, a protein normally present on red blood cells. Flow cytometry of blood to detect low levels of CD59 and CD55 has now replaced Ham’s test as the gold standard investigation in PNH.

179
Q

A study looks at the use of bisphosphonates in controlling the pain associated with bone metastases. 120 patients are enrolled in the study, 40 are given conventional treatment with NSAIDs and radiotherapy. Of the 80 patients who were given bisphosphonates, 40 received significant pain relief. What is the odds of a patient with bone metastases receiving significant pain relief from bisphosphonates?
A) 0.33
B) 3
C) 2
D) 1
E) 0.5

A

Odds are a ratio of the number of people who incur a particular outcome to the number of people who do not incur the outcome.

40 of the 80 patients received significant pain relief

It can therefore be inferred that 40 of the 80 patients did not receive significant pain relief.

Therefore the odds are 40 / 40 = 1

180
Q

A 56-year-old Muslim man with a history of type 2 diabetes asks for advice. He is due to start fasting for Ramadan and is unsure what he should do with regards to his diabetes medications. He currently takes metformin 500mg tds. What is the most appropriate advice?
A) Switch to S/C biphasic insulin
B) 500mg at predawn meal, 1000mg at sunset meal
C) No change to metformin dose
D) 1000mg at predawn meal, 500mg at sunset meal
E) Stop metformin for duration of Ramadan

A

During Ramadan, one-third of the normal metformin dose should be taken before sunrise and two-thirds should be taken after sunset.

181
Q

An 18M presents with a third episode of balanitis over the past 3 months. He also has vague symptoms of tiredness. His father and grandfather were diagnosed with type 1 diabetes and take a basal-bolus insulin regimen. He is slim with a body mass index of 22 kg/m². He is noted to have glycosuria on urine dipstick testing. His fasting blood glucose yields a result of 9.1mmol/l. What is the most likely diagnosis?
A) Maturity onset diabetes of the young
B) Latent autoimmune diabetes of adults
C) T1DM
D) T2DM
E) Renal glycosuria

A

MODY is autosomal dominant diabetes mellitus which presents for the first time in young slim individuals without symptoms of polyuria and polydipsia. Many patients with MODY including those with the HNF-1 alpha form of the disease, can be managed with sulphonylureas for years before needing to start insulin therapy. Evaluation of family history and testing for antibodies for type 1 diabetes can help to differentiate MODY from other forms of diabetes mellitus.

MODY is characterised by the development of type 2 diabetes mellitus in patients < 25 years old.

Type 1 diabetes isn’t associated with such strong heritability and given this patient’s body habitus, type 2 diabetes is very unlikely. LADA is associated with a body habitus similar to the overweight / obese picture seen in type 2 diabetes for many patients, although progression to insulin therapy occurs more quickly.

182
Q

A 65M presents with haemoptysis. He has no past medical history. He smokes 20 cigarettes per day. He is found to have an AKI and ++blood and ++protein on urinalysis. A chest x-ray demonstrates multifocal airspace opacification. A renal biopsy demonstrates IgG deposits. What serological test is most likely to be positive?
A) Anti ds-DNA
B) Anti-GBM
C) cANCA
D) pANCA
E) IgA

A

Goodpasture’s syndrome
-IgG deposits on renal biopsy
-Anti-GBM antibodies

ANCA vasculitis can cause a pulmonary-renal syndrome that is indistinguishable clinically from anti-GBM disease. However, the renal biopsy in ANCA vasculitis would demonstrate vasculitis rather than the deposition of IgG antibodies. c-ANCA is most likely to be associated with proteinase-3 antibodies and clinically may also have ear, nose and throat (ENT) disease.

183
Q

Which of the following deficiencies are most likely to be associated with recurrent meningococcal meningitis infections?
A) C1
B) C3
C) C4
D) C5
E) C59

A

Deficiencies of the membrane attack complex, (C5-C9) are particularly associated with meningococcal infection.

Deficiencies of the C1 complex are recognised to be closely associated with the development of systemic lupus erythematosus. C3 deficiency is associated with a predisposition to infection with capsulated bacterial infections such as pneumococcus and Haemophilus which begins quite soon after birth. C4 deficiency is rare and is associated with early onset systemic lupus erythematosus. CD59 deficiency is involved in the pathogenesis of paroxysmal nocturnal haemoglobinuria.

184
Q

A 52M has been referred due to the development of difficultly in finding the right words whilst speaking. His comprehension of normal conversation has however remained normal. Where is the likely lesion?
A) Anterior temporal lobe
B) Posterior temporal lobe
C) Parietal lobe
D) Lateral part of frontal lobe (inferior frontal gyrus)
E) Medial part of frontal lobe (cingulate gyrus)

A

This man has expressive aphasia due to a lesion in Broca’s area, located on the posterior aspect of the frontal lobe, in the inferior frontal gyrus. Speech here would be non-fluent, laboured and halting.

Temporal lobe lesion may lead to Wernicke’s aphasia: this area ‘forms’ the speech before ‘sending it’ to Brocas area. Lesions result in word substituion, neologisms but speech remains fluent.

185
Q

What is the mechanism of action of sitagliptin?
A) Decreases GLP breakdown
B) Decreases levels of GiP incretins
C) Increases GLP production
D) Increases insulin secretion from the pancreas
E) Mimics GLP-1

A

By blocking the DPP-4 enzyme, sitagliptin reduces the peripheral breakdown of GLP-1 and GIP. This potentiates the incretin effect which is typically reduced in diabetes.

Exenatide and liraglutide are examples of GLP-1 mimetics. They increase insulin secretion and inhibit glucagon secretion. One of the major advances of GLP-1 mimetics is that they typically result in weight loss, in contrast to insulin, sulfonylureas and thiazolidinediones. They are sometimes used in combination with insulin in T2DM to minimise weight gain.

186
Q

A 24F presents due to increased frequency of migraine attacks. She is now having around four migraines per month. Which type of medication would it be most appropriate to prescribe to reduce the frequency of migraine attacks?
A) 5-HT2 agonist
B) 5-HT1 antagonist
C) TCA
D) B-blocker
E) 5-HT1 agonist

A

Topiramate is also recommended by NICE as first-line prophylaxis against migraine. However, given that she is female and of child-bearing age a beta-blocker (such as propranolol) is a better choice.

187
Q

A 65M is referred to the neurology outpatient clinic due to a resting tremor of his right hand. A diagnosis of Parkinson’s disease is made. He is otherwise well and is not currently disabled by his symptoms. What is the most appropriate treatment?
A) Amantadine
B) No treatment needed
C) New generation dopamine receptor agonist (ropinorole)
D) Conventional dopamine receptor agonist (bromocriptine)
E) Antimuscarinics

A

Following the NICE 2017 guidelines, it is now recommended to use new generation dopamine receptor agonist for motor symptoms that are not affecting a patient’s quality of life.

For first-line treatment:
-If the motor symptoms are affecting the patient’s quality of life: levodopa
-If the motor symptoms are not affecting the patient’s quality of life: dopamine agonist (non-ergot derived), levodopa or monoamine oxidase B (MAO‑B) inhibitor

188
Q

What is the most appropriate management of a patient with medication overuse headaches following the use of paracetamol 1g QDS and ibuprofen 400mg TDS every day?
A) Add metoclopramide and start propranolol
B) Gradually withdraw analgesics and start propranolol
C) Abruptly stop analgesics
D) Gradually withdraw analgesics
E) Continue analgesics and start propranolol

A

Medication overuse headache
-simple analgesia + triptans: stop abruptly
-opioid analgesia: withdraw gradually

189
Q

Which of the following medications can be used to treat acute mountain sickness?
A) Acetazolamide
B) Betahistine
C) Cyclizine
D) Dexamethasone
E) Nifedipine

A

Prevention of acute mountain sickness includes limiting gain of altitude to 500m/day, increasing physical fitness and acetazolamide, a carbonic anhydrase inhibitor that causes metabolic acidosis and a compensatory respiratory alkalosis to increase respiratory rate and tissue oxygenation.

190
Q

A 62–year-old woman attends with a chronic productive cough and shortness of breath on exertion. She has never smoked. A CT scan indicates abnormal widening and thickening of the bronchi; consolidation of the right lower lobe. What is the most likely organism that will be cultured?
A) Haemophilus influenza
B) Klebsiella oxytoca
C) Klebsiella pneumonia
D) Pseudomonas aeruginosa
E) Streptococcus pneumoniae

A

Bronchiectasis: most common organism = Haemophilus influenzae

191
Q

Which of the following is most associated with G6PD deficiency?
A) Blister cells
B) Burr cells
C) Target cells
D) Schistocytes
E) Sickle cells

A

In G6PD deficiency, blood film may show bite cells, blister cells, and Heinz bodies.

Schistocytes are seen in MAHA (e.g., DIC, HUS). DIC can cause bleeding from various sites, while HUS, along with haemolytic anaemia, presents with thrombocytopenia and renal failure.

192
Q

Which of the following cytokines are inhibited by calcineurin inhibitors?
A) IL1
B) IL2
C) IL6
D) IL10
E) TNFa

A

Calcineurin inhibitors inhibit calcineurin - the activity of which induces IL-2. Reduction of IL-2 leads to less T-cell mediated transplant rejection.

193
Q

What is the most likely effect of excess aspirin on acid-base balance?
A) Metabolic acidosis
B) Metabolic alkalosis
C) Normal pH
D) Respiratory acidosis
E) Respiratory alkalosis

A

Salicylates directly stimulate the respiratory centre to produce an increase in depth and rate of breathing, thereby causing a respiratory alkalosis. Metabolic acidosis is a characteristic feature as the toxicity progresses and is not solely a compensatory mechanism.

194
Q

Which of the following is the most specific test for SLE?
A) Anti-Smith antibody
B) Anti-centromere antibody
C) Anti-histone antibody
D) ANCA
E) ANA

A

The most sensitive test for SLE is ANA with 100% sensitivity. However, there are other tests for SLE with better specificity. The most specific tests are anti-dsDNA antibody and an anti-Smith antibody with 99% specificity.

Anti-histone antibodies are present in more than 95% of patients with drug-induced lupus, especially among those taking procainamide, hydralazine, chlorpromazine, and quinidine.

195
Q

Which of the following is the most characteristic finding on CXR of Goodpastures disease?
A) B/L diffuse infiltrates in the lower zones
B) Patchy shadows in apical zones
C) B/L hilar lymphadenopathy
D) Lobar consolidation
E) Pleural effusion

A

In Goodpastures disease, B/L diffuse infiltrates in the lower zones are characteristic and represent pulmonary haemorrhages.

196
Q

Which of the following is the blood supply to the terminal ileum?
A) Ileocolic artery
B) Inferior pancreaticoduodenal artery
C) Marginal artery
D) Middle colic artery
E) Renal colic artery

A

Ileocolic artery - supplies blood to terminal artery and appendix - it is the most inferior branch of the SMA

197
Q

Which of the following vaccines should be avoided in a patient with severe egg allergy?
A) Hepatitis A
B) Hepatitis B
C) MMR
D) Recombinant influenza vaccine
E) Yellow fever

A

Yellow fever vaccine contains significant amounts of egg protein.

198
Q

Which of the following pH results is most likely to be seen on aspiration if an NGT is in the right place?
A) <1.5
B) 1.5-3.5
C) 3.5-5.5
D) 5.5-7.5
E) >7.5

A

Gastric juice is normally at a pH range between 1.5 and 3.5

199
Q

Which of the following is the appropriate intervention for a patient with homocysteinuria?
A) Ascorbic acid
B) Homocysteine exclusion
C) Niacin
D) Phenylalanine exclusion
E) Pyridoxine

A

50% of patients are responsive to pyridoxine supplementation.

Phenylalanine exclusion is the treatment of choice for phenylketonuria.

200
Q

A patent is admitted with symptoms suggestive of Wernicke’s encephalopathy. He is found to have a blood glucose of 2.9. What is the most appropriate management?
A) IV 5% dextrose
B) IV 20% dextrose
C) IV Vitamin B12
D) IV glucagon
E) IV thiamine

A

IV 20% dextrose should be given to correct the hypoglycaemia prior to treatment with IV thiamine.

201
Q

Which of the following is most likely to be found in a patient with primary pulmonary hypertension?
A) LVH
B) Left atrial dilatation
C) MR
D) PS
E) TR

A

The extensive back pressure from pulmonary hypertension can lead to TR which, in time, can lead to left atrial dilatation.

202
Q

A patient presents with sudden onset R hemiplegia and L third nerve palsy. Which of the following cerebral arteries are most likely to have been occluded?
A) Anterior cerebral
B) Anterior communicating
C) Middle cerebral
D) Posterior cerebral
E) Posterior inferior cerebellar

A

Posterior cerebral - ipsilateral third nerve palsy and contralateral hemiplegia is characteristic of Weber syndrome.

203
Q

Which of the following is the most sensitive way to differentiate acute from chronic renal impairment?
A) Hb
B) Serum Ca
C) PTH
D) Phosphate
E) Urinary protein

A

Serum PTH

204
Q

Which of the following signs would most suggest cardiac tamponade?
A) Anterior chest pain
B) Bradycardia
C) Raised JVP
D) Elevated troponin
E) Pulsus paradoxes

A

SOB, tachycardia and a raised JVP are often seen in cardiac tamponade.

205
Q

What is the MOA of rasburicase?
A) HGPT inhibition
B) Tubular disruption
C) Urate transporter 1 inhibition
D) Urate oxidation
E) Xanthine oxidase inhibition

A

Rasburicase is recombinant urate oxidase that drives the metabolism of uric acid.

206
Q

How long does it take before the risk of splenic rupture is back to normal levels following EBV infection?
A) 2W
B) 4W
C) 8W
D) 12W
E) 24W

A

4W - NICE guidelines currently state to avoid contact sport for 4W following EBV diagnosis

207
Q

Which of the following is the initial intervention of choice for a patient with non-falciparum malaria?
A) Artemether and lumefantrine
B) Artesunate
C) Chloroquine
D) Doxycycline
E) Quinine and doxycycline

A

Artemether and lumefantrine is the initial intervention of choice for non-falciparum malaria. Primaquine should also overlap with this initial therapy to prevent relapse.

Artesunate is the intervention of choice for patients with severe falciparum malaria.

208
Q

Which one of the following types of oral steroid has the least amount of mineralocorticoid activity?
A) Fludocortisone
B) Hydrocortisone
C) Dexamethasone
D) Prednisolone
E) Cortisone

A

Minimal glucocorticoid activity, very high mineralocorticoid activity - Fludrocortisone

Glucocorticoid activity, high mineralocorticoid activity - Hydrocortisone

Predominant glucocorticoid activity, low mineralocorticoid activity - Prednisolone

Very high glucocorticoid activity, minimal mineralocorticoid activity - Dexamethasone, Betamethasone

209
Q

What is the mechanism of transmission of C difficile infection?
A) Aerosol
B) Droplet
C) Faeco-oral by ingestion of bacteria
D) Faeco-oral by ingestion of spores
E) Waterborne

A

Clostridioides difficile: spreads via the faecal-oral route by ingestion of spores

210
Q

Which of the following adverse effects should a patient started n cyclophosphamide be warned about?
A) Cataracts
B) Open-angle glaucoma
C) Retinopathy
D) Scleritis
E) Uveitis

A

A recognised side effect of this drug is ‘bull’s eye retinopathy’, which may result in severe and permanent visual loss.